Preview (15 of 95 pages)

HESI EXIT RN EXAM V1-V7 2023
Version 1
Part 2
361. A female nurse who took drugs from the unit for personal use was temporarily released
from duty. After completion of mandatory counseling, the nurse has asked administration to
allow her to return to work. When the nurse administrator approaches the charge nurse with
the impaired nurse request, which action is best for the charge nurse to take?
A. Since treatment is completed, assign the nurse to the route RN responsibilities
B. Ask to meet with impaired nurse’s therapist before allowing her back on the unit.
C. Allow the impaired nurse to return to work and monitor medication administration
D. Meet with staff to assess their feelings about the impaired nurse’s return to the unit.
Answer: C. Allow the impaired nurse to return to work and monitor medication
administration
Rationale:
Provides essential monitoring and helps ensure nurse compliance and promote client safety.
362. In making client care assignment, which client is best to assign to the practical nurse
(PN) working on the unit with the nurse?
A. An immobile client receiving low molecular weight heparin q12 h.
B. A client who is receiving a continuous infusion of heparin and gets out of bed BID
C. A client who is being titrated off heparin infusion and started on PO warfarin (Coumadin)
D. An ambulatory client receiving warfarin (Coumadin) with INR of 5 second.
Answer: A. An immobile client receiving low molecular weight heparin q12 h.
Rationale:
A describe the most stable client. The other ones are at high risk for bleeding problems and
require the assessment skills.
363. A client who is admitted to the intensive care unit with a right chest tube attached to a
THORA-SEAL chest drainage unit becomes increasingly anxious and complain of difficulty
breathing. The nurse determine the client is tachypneic with absent breath sounds in the
client’s right lungs fields. Which additional finding indicates that the client has developed a
tension pneumothorax?

A. Continuous bubbling in the water seal chamber
B. Decrease bright red blood drainage
C. Tachypnea and difficulty breathing
D. Tracheal deviation toward the left lung.
Answer: D. Tracheal deviation toward the left lung.
Rationale:
Tracheal deviation toward the unaffected left lung with absent breath sounds over the affected
right lung are classic late signs of a tension pneumothorax.
364. A low-risk primigravida at 28-weeks gestation arrives for her regular antepartal clinic
visit. Which assessment finding should the nurse consider within normal limits for this client?
A. Pulse increase of 10 beats/minute
B. Proteinuria
C. Glucosuria
D. Fundal height 0f 22 centimeters
Answer: A. Pulse increase of 10 beats/minute
365. The nurse discovers that an elderly client with no history of cardiac or renal disease has
an elevated serum magnesium level. To further investigate the cause of this electrolyte
imbalance, what information is most important for the nurse to obtain from the client’s
medical history?
Answer: Frequency of laxative use for chronic constipation
366. Which action should the nurse implement with auscultating anterior breath sounds?
(Place the first action on top and last action on the bottom)
1. Place stethoscope in suprasternal area to auscultate for bronchial sounds
2. Auscultate bronchovesicular sounds from side to side the first and second intercostal
spaces
3. Displace female breast tissue and apply stethoscope directly on chest wall to hear vesicular
sounds
4. Document normal breath sounds and location of adventitious breath sounds
Answer: 1. Place stethoscope in suprasternal area to auscultate for bronchial sounds
2. Auscultate bronchovesicular sounds from side to side the first and second intercostal
spaces

3. Displace female breast tissue and apply stethoscope directly on chest wall to hear vesicular
sounds
4. Document normal breath sounds and location of adventitious breath sounds
367. A client with chronic alcoholism is admitted with a decreased serum magnesium level.
Which snack option should the nurse recommend to this client?
A. Cheddar cheese and crackers.
B. Carrot and celery sticks.
C. Beef bologna sausage slices.
D. Dry roasted almonds.
Answer: D. Dry roasted almonds.
Rationale:
Alcoholism promotes inadequate food intake and gastrointestinal loss of magnesium include
green leafy vegetables and nuts and seeds. Other snacks listed provide much lower amounts
of magnesium per serving.
368. The nurse is preparing a teaching plan for an older female client diagnosed with
osteoporosis. What expected outcome has the highest priority for this client?
A. Identifies 2 treatments for constipation due to immobility.
B. Names 3 home safety hazards to be resolve immediately.
C. State 4 risk factors for the development of osteoporosis.
D. Lists 5 calcium-rich foods to be added to her daily diet.
Answer: B. Names 3 home safety hazards to be resolve immediately.
Rationale:
A major teaching goal for an elderly client with osteoporosis is maintenance of safety to
prevent falls. Injury due to a fall, usually resulting in a hip fracture, can result in reduced
mobility and associated complications. Other goals are also important when teaching clients
who have osteoporosis, but they do not have the priority of preventing falls, which relates to
safety.
369. The nurse is teaching a male adolescent recently diagnosed with type 1diabetes mellitus
(DM) about self-injecting insulin. Which approach is best for the nurse to use to evaluate do
you effectiveness of the teaching?
A. Ask the adolescent to describe his level of comfort with injecting himself with insulin.

B. Observe him as he demonstrates self-injection technique in another diabetic adolescent
C. Have the adolescent list the procedural steps for safe insulin administration.
D. Review his glycosylated hemoglobin level 3 months after the teaching session.
Answer: B. Observe him as he demonstrates self-injection technique in another diabetic
adolescent
Rationale:
Watching the adolescent perform the procedure with another adolescent provides peer support
the most information regarding his skill with self-injection. Other options do not provide
information about the effectiveness of nurse’s teaching.
370. A young adult woman visits the clinic and learns that she is positive for BRCA1 gene
mutation and asks the nurse what to expect next. How should the nurse respond?
A. Explain that counseling will be provided to give her information about her cancer risk
B. Gather additional information about the client’s family history for all types of cancer.
C. Offer assurance that there are a variety of effective treatments for breast cancer.
D. Provide information about survival rates for women who have this genetic mutation.
Answer: A. Explain that counseling will be provided to give her information about her
cancer risk
Rationale:
BRACA1or BRACA2 genetic mutation indicates an increased risk for developing breast or
ovarian cancer and genetic counseling should be provided to explain the increased risk (A)to
the client along with options for increased screening or preventative measures. (B) Is
completed by the genetic counsellor before the client undergoes genetic testing. a positive
BRACA1test is not an indicator of the presence of cancer and (C and D) are not appropriate
responses prior to genetic counseling.
371. A mother runs into the emergency department with s toddler in her arms and tells the
nurse that her child got into some cleaning products. The child smells of chemicals on hands,
face, and on the front of the child's clothes. After ensuring the airway is patent, what action
should the nurse implement first?
A. Call poison control emergency number.
B. Determine type of chemical exposure.
C. Obtain equipment for gastric lavage.
D. Assess child for altered sensorium.

Answer: B. Determine type of chemical exposure.
Rationale:
Once the type of chemical is determined, poison control should be called even if the chemical
is unknown. If lavage is recommended by poison control, intubation and nasogastric tube
may be needed as directed by poison control. Altered sensorium, such as lethargy, may occur
if hydrocarbons are ingested
372. The nurse assigned unlicensed assistive personnel (UAP) to apply antiembolism
stockings to a client. The nurse and UAP enters the room, the nurse observes the stockings
that were applying by the UAP. The UAP states that the client requested application of the
stockings as seen on the picture, for increased comfort. What action should the nurse take?
A. Ask the client if the stocking feel comfortable.
B. Supervise the UAP in the removal of the stockings.
C. Place a cover over the client’s toes to keep them warm.
D. Discussed effective use of the stockings with the client on UAP
Answer: D. Discussed effective use of the stockings with the client on UAP
Rationale:
Antiembolism stockings are designed to fit securely and should be applied so that there are
no bands of the fabric constricting venous return. The nurse should discuss the need for
correct and effective use of the stockings with both the client and UAP to improve
compliance. Other options do not correct the incorrect application of the stockings.
373. Nurses working on a surgical unit are concerned about the physicians treatment of
clients during invasive procedures, such as dressing changes and insertion of IV lines. Clients
are often crying during the procedures, and the physician is usually unconcerned or annoyed
by the client’s response. To resolve this problem, what actions should the nurses take?
(Arrange from the first action on the top of the list on the bottom)
1. Talk to the physician as a group in a non-confrontational manner.
2. Document concerns and report them to the charge nurse.
3. Submit a written report to the director of nursing.
4. Contact the hospital’s chief of medical services.
5. File a formal complaint with the state medical board.
Answer: 2. Document concerns and report them to the charge nurse.
1. Talk to the physician as a group in a non-confrontational manner.

3. Submit a written report to the director of nursing.
4. Contact the hospital’s chief of medical services.
5. File a formal complaint with the state medical board.
Rationale:
Nurses have both an ethical and legal responsibility to advocate for clients’ physical and
emotional safety. Talking with the physician in a nonconfrontational manner is the first step
in conflict resolution. If this is not effective, the organizational chain of ineffective, a formal
complaint with the state medical board should be implemented.
374. While changing a client’s chest tube dressing, the nurse notes a crackling sensation when
gentle pressure is applied to the skin at the insertion site. What is the best action for the nurse
to take?
A. Apply a pressure dressing around the chest tube insertion site.
B. Assess the client for allergies to topical cleaning agents.
C. Measure the area of swelling and crackling.
D. Administer an oral antihistamine per PRN protocol.
Answer: C. Measure the area of swelling and crackling.
Rationale:
A crackling sensation, or crepitus, indicates subcutaneous emphysema, or air leaking into the
skin. This area should be measured and the finding documented. Other options are not
indicated for crepitus.
375. To prevent infection by auto contamination during the acute phase of recovery from
multiple burns, which intervention is most important for the nurse to implement?
A. Dress each wound separately.
B. Avoid sharing equipment between multiple clients.
C. Use gown, mask and gloves with dressing change.
D. Implement protective isolation.
Answer: A. Dress each wound separately.
Rationale:
Each wound should be dressed separately using a new pair of sterile glove to avoid auto
contamination (the transfer of microorganisms form one infected wound to a non-infected
wound). The other choices do not prevent auto contamination.

376. The nurse is preparing an intravenous (IV) fluid infusion using an IV pump.
Within 30 seconds of turning on the machine, the pump’s alarm beeps “occlusion”. What
action should the nurse implement first?
A. Flush the vein with 3 ml of sterile normal saline.
B. Assess the IV catheter insertion site for infiltration.
C. Verify the threading of the tubing through the IV pump.
D. Determine if the clamp on the IV tubing is released
Answer: D. Determine if the clamp on the IV tubing is released
Rationale:
When the pump immediately beeps, it is often because the IV tubing clamp is occluding the
flow, so the clamp should be checked first to ensure that it is open. If the alarm is not
eliminated after the tubing clamp is released, flushing the IV site with saline is a common
practice to clean the needle or to identify resistance due to another source. Local signs of
infiltration may indicate the need to select another vein, but the pump’s beeping-this early in
the procedure is likely due to a mechanical problem. If beeping continues after verifying that
the clamp is released the placement or threading of the tubing through the pump should be
verified.
377. A client with arthritis has been receiving treatment with naproxen and now reports
ongoing stomach pain, increasing weakness, and fatigue. Which laboratory test should the
nurse monitor?
A. Sed rate (ESR)
B. Haemoglobin
C. Calcium
D. Osmolality.
Answer: B. Haemoglobin
Rationale:
Naproxen can cause gastric bleeding, so the nurse should monitor the client’s haemoglobin to
assess for possible bleeding. Other options are not likely to be affected by the used of
naproxen and are not related to the client’s current symptoms.
378. The nurse assesses a child in 90-90 traction. Where should did nurse assess for signs of
compartment syndrome?
Answer:

Rationale:
Compartment syndrome is the result of swelling and subsequent reduction in circulation to
the area distal to the compartment. This can be a complication of traumatic injury and cast
administration, so it is important to assess circulation distal to the casted prolonged capillary
refill.
379. After receiving the Braden scale findings of residents at a long-term facility, the charge
nurse should to tell the unlicensed assistive personnel (UAP) to prioritize the skin care for
which client?
A. An older adult who is unable to communicate elimination needs.
B. An older man whose sheets are damped each time he is turned.
C. A woman with osteoporosis who is unable to bear weight.
D. A poorly nourished client who requires liquid supplement.
Answer: B. An older man whose sheets are damped each time he is turned.
Rationale:
A Braden score of less than 18 indicates a risk for skin breakdown, and clients with such
score require intensive nursing care. Constant moisture places the client at a high risk for skin
breakdown, and interventions should be implemented to pull moisture away from the client’s
skin. Other options may be risk factors but do not have as high a risk as constant exposure to
moisture.
380. A client with acute renal failure (ARF) is admitted for uncontrolled type 1 diabetes
Mellitus and hyperkalemia. The nurse administers an IV dose of regular insulin per sliding

scale. Which intervention is the most important for the nurse to include in this client’s plan of
care?
A. Monitor the client’s cardiac activity via telemetry.
B. Maintain venous access with an infusion of normal saline.
C. Assess glucose via fingerstick q4 to 6 hours.
D. Evaluate hourly urine output for return of normal renal function.
Answer: A. Monitor the client’s cardiac activity via telemetry.
Rationale:
As insulin lowers the blood glucose of a client with diabetic ketoacidosis (DKA), potassium
returns to the cell but may not impact hyperkalemia related to acute renal failure. The priority
is to monitor the client for cardiac dysrhythmias related to abnormal serum potassium levels.
IV access, assessment of glucose level, and monitoring urine output are important
interventions, but do not have the priority of monitoring cardiac function.
381. A client with C-6 spinal cord injury rehabilitation. In the middle of the night the client
reports a severe, pounding headache, and has observable piloerection or “goosebumps”. The
nurse should asses for which trigger?
A. Loud hallway noise.
B. Fever
C. Full bladder
D. Frequent cough.
Answer: C. Full bladder
Rationale:
A pounding headache is a sign of autonomic hyperreflexia, an acute emergency that occurs
because of an exaggerated sympathetic response in a client with a high level spinal cord
injury. Any stimulus below the level of injury can trigger autonomic hyperreflexia, but the
most common cause is an overly distended bladder. The other options are unlikely to produce
the manifestation of autonomic hyperreflexia.
382. A nurse working on an endocrine unit should see which client first?
A. An adolescent male with diabetes who is arguing about his insulin dose.
B. An older client with Addison’s disease whose current blood sugar level is 62mg/dl (3.44
mmol/l).

C. An adult with a blood sugar of 384mg/dl (21.31mmol/l) and urine output of 350 ml in the
last hour.
D. A client taking corticosteroids who has become disoriented in the last two hours.
Answer: D. A client taking corticosteroids who has become disoriented in the last two hours.
Rationale:
Meeting the client’s need for safety is a priority intervention. Mania and psychosis can occur
during corticosteroids therapy, places the client at risk for injury, so the patient taking
corticosteroids should be seen first.
383. A client is receiving and oral antibiotic suspension labelled 250 mg/2ml. The healthcare
provider prescribes 200mg every 6 hours. How many ml should the nurse administer at each
dose? (Enter numerical value only. If rounding is required, round to the nearest tenth)
Answer: 1.6
Rationale:
Using the formula D/H x Q
200mg/250 mg x 2ml = 200/250 = 1.6 ml
384. Four hours after surgery, a client reports nausea and begins to vomit. The nurse notes
that the client has a scopolamine transdermal patch applied behind the ear. What action
should the nurse take?
A. Reposition the transdermal patch to the client’s trunk.
B. Remove the transdermal patch until the vomiting subsides.
C. Notify the healthcare provider of the vomiting.
D. Explain that this is a side effect of the medication in the patch.
Answer: C. Notify the healthcare provider of the vomiting.
Rationale:
Transdermal scopolamine is used to prevent nausea and vomiting from anaesthesia and
surgery. The nurse should notify the healthcare provider if the medication is ineffective. The
patch should be applied behind the ear and should remain in place to reduce the nausea and
vomiting. Nausea and vomiting are no side effects of the medication.
385. The nurse identifies an electrolyte imbalance, an elevated pulse rate, and elevated BP for
a client with chronic kidney disease. Which is the most important action for the nurse to take?
A. Monitor daily sodium intake.

B. Record usual eating patterns.
C. Measure ankle circumference.
D. Auscultate for irregular heart rate.
Answer: D. Auscultate for irregular heart rate.
Rationale:
Chronic kidney failure (CKF) is a progressive, irreversible loss of kidney functions,
decreasing glomerular filtration rate (GFR), and the kidney’s inability to excrete metabolic
waste products and water, resulting in fluid overload, elevated pulse, elevated BP and
electrolytes imbalances. The most important action for the nurse to implement is to auscultate
for irregular heart rate (D) due to the decreased excretion of potassium by the kidneys. (A, B,
and C) are not as important as monitoring for fatal cardiac dysrhythmias related to
hyperkalaemia.
386. A client with persistent low back pain has received a prescription for electronic
stimulator (TENS) unit. After the nurse applies the electrodes and turns on the power, the
client reports feeling a tingling sensation. How should the nurse respond?
A. Determine if the sensation feels uncomfortable.
B. Decrease the strength of the electrical signals.
C. Remove electrodes and observe for skin redness.
D. Check the amount of gel coating on the electrodes.
Answer: A. Determine if the sensation feels uncomfortable.
Rationale:
Electronic stimulators, such as a trans electrical nerve stimulator (TENS) unit, have been
found to be effective in reducing low back pain by “closing the gate” to pain stimuli. A
tingling sensation should be felt when the power is turned on, and the nurse should assess
whether the sensation is too strong, causing discomfort or muscle twitching. Decreasing the
electrical signal may be indicated if the sensation is too strong. Other options are not
necessary because the tingling sensation is expected.
387. A female client is extremely anxious after being informed that her mammogram was
abnormal and needs to be repeated. Client is tearful and tells the nurse her mother died of
breast cancer. What action should the nurse take?
A. Provide the client with information about treatment options for breast cancer.
B. Reassure the client that the final diagnosis has not been made.

C. Encourage the client to continue expressing her fears and concerns.
D. Suggest to the client that she seek a second opinion.
Answer: C. Encourage the client to continue expressing her fears and concerns.
Rationale:
The nurse should show support for the client by encouraging her to continue expressing her
concerns. A diagnosis has not yet been made, so it is too early to discuss treatment options.
Other options dismiss the client’s feelings or are premature given that the diagnosis is not yet
made.
388. The psychiatric nurse is talking to a newly admitted client when a male client diagnosed
with antisocial behavior intrudes on the conversation and tells the nurse, “I have to talk to
you right now! It is very important!” how should the nurse respond to this client?
A. Put his behavior on extinction and continue talking with the newly admitted.
B. Inform him that the nurse is busy admitting a new client and will talk to him later.
C. Encourage him to go to the nurse’s station and talk with another nurse.
D. Introduce him to the newly admitted client and ask him to him to join in the conversation.
Answer: B. Inform him that the nurse is busy admitting a new client and will talk to him
later.
Rationale:
The psychiatric nurse must set limits with antisocial behavior so that appropriate behavior is
demonstrated. Interrupting a conversation is rude and inappropriate, so telling the client that
they can talk later is the best course of action. Other options may cause the client to become
angry and they do not address the client’s behavior. The nurse should not involve this client
with newly admitted client’s admission procedure.
389. The charge nurse is planning for the shift and has a registered nurse (RN) and a practical
nurse (PN) on the team. Which client should the charge nurse assign to the RN?
A. A 64 year old client who had a total hip replacement the previous day.
B. A 75 year old client with renal calculi who requires urine straining.
C. An adolescent with multiple contusions due to a fall that occurred 2 days ago.
D. A 30 year old depressed client who admits to suicide ideation.
Answer: D. A 30 year old depressed client who admits to suicide ideation.
Rationale:

A client who is suicidal requires psychological assessment, therapeutic communication and
knowledge beyond the educational level of a practical nurse (RN). Other clients could be
cared for by the PN or the UAP, with supervision by the registered nurse.
390. A female client presents in the Emergency Department and tells the nurse that she was
raped last night. Which question is most important for the nurse to ask?
A. Does she knows the person who raped her?
B. Has she taken a bath since the raped occurred?
C. Is the place where she lived a safe place?
D. Did she report the rape to the police Department?
Answer: B. Has she taken a bath since the raped occurred?
Rationale:
The priority action is collected the forensic evidence, so asking if the has taken a bath since
the rape occurred is the most important information to obtain. Other options are used by law
enforcement to determine the perpetrator and are not vital in providing client care at this time.
391. While caring for a client’s postoperative dressing, the nurse observes purulent drainage
at the wound. Before reporting this finding to the healthcare provider, the nurse should
review which of the client’s laboratory values?
A. Serum albumin
B. Creatinine level
C. Culture for sensitive organisms.
D. Serum blood glucose (BG) level
Answer: C. Culture for sensitive organisms.
Rationale:
A client who has a postoperative dressing with purulent drainage from the wound is
experiencing an infection. The nurse should review the client’s laboratory culture for
sensitive organisms (C) before reporting to the healthcare provider. (A, B and D) are not
indicated at this time.
392. The nurse is demonstrating correct transfer procedures to the unlicensed assisted
personnel (UAP) working on a rehabilitation unit. The UAPs ask the nurse how to safely
move a physically disabled client from the wheelchair to a bed. What action should the nurse
recommended?

A. Hold the client at arm’s length while transferring to better distribute the body weight.
B. Apply the gait belt around the client’s waits once standing position has been assumed.
C. Place a client’s locked wheelchair on the client’s strong side next to the bed.
D. Pull the client into position by reaching from the opposite side of the bed.
Answer: C. Place a client’s locked wheelchair on the client’s strong side next to the bed.
Rationale:
Placing the wheelchair on the client’s strong side offers the greatest stability for the transfer.
Holding the client arm’s length or pulling from the opposite site of the bed reflect poor body
mechanism. Using a gait belt offers additional safety for the client, but should be done after
the wheelchair has been put into the proper place and the wheels have been locked and before
the client has assumed a standing position.
393. A client who is experiencing musculoskeletal pain receives a prescription for ketorolac
15mg IM q6 hours. The medication is depended in a 39mg/ml pre-filled syringe. Which
action should the nurse implement when giving the medication?
A. Administer the entire pre-filled syringe deep in the dorsogluteal site.
B. Use a separate syringe to remove 15mg from the pre-filled syringe and give in the back of
the arm.
C. Waste 0.5 ml from the pre-filled syringe and inject the medication in the ventrogluteal site.
D. Call the healthcare provider to request a prescription change to match the dispensed 30mg
dose.
Answer: C. Waste 0.5 ml from the pre-filled syringe and inject the medication in the
ventrogluteal site.
Rationale:
The pre-filled contain 30mg /1ml, so 0.5ml should be wasted to obtain the correct dosage of
15mg for administration in the preferred IM ventrogluteal site. The nurse is responsible for
calculating and preparing the prescribed dose using the available concentration, so other
options are not indicated.
394. A client with a lower respiratory tract infection receives a prescription for ciprofloxacin
500mg PO q 12hours. When the client request an afternoon snack, which dietary choice
should the nurse provide?
A. Vanilla-flavoured yogurt
B. Low fat chocolate milk.

C. Calcium fortified juice
D. Cinnamon applesauce
Answer: D. Cinnamon applesauce
Rationale:
Dairy products and calcium fortified dairy products decrease the absorption of ciprofloxacin.
Cinnamon applesauce contains no calcium, so this is the best snack selection. Since other
options contains calcium, these snack should be avoided by a client who is taking
ciprofloxacin.
395. The healthcare provider prescribes a low-fibre diet for a client with ulcerative colitis.
Which food selection would indicate to the nurse the client understands they prescribed diet?
A. Roasted turkey canned vegetables
B. Baked potatoes with skin raw carrots
C. Pancakes whole-grain cereal's
D. Roast pork fresh strawberries
Answer: A. Roasted turkey canned vegetables
Rationale:
Foods allowed on a low-fibre diet includes roasted or baked turkey and canned vegetables the
foods in the other options are not low in fibre
396. An adult client with schizophrenia begin treatment three days ago with the Antipsychotic
risperidone. The client also received prescription for trazodone as needed for sleep and
clonazepam as needed for severe anxiety. When the client reports difficulty with swallowing,
what action should the nurse take?
A. Obtain a prescription for an anticholinergic medication
B. Determine how many hours declined slept last night
C. Administer the PRN prescription for severe anxiety
D. Watch the thyroid cartilage move while the client swallows
Answer: A. Obtain a prescription for an anticholinergic medication
Rationale:
Antipsychotic medications have an extrapyramidal side effects one of which is difficult to
swallowing the nurse should obtain a prescription for an anticholinergic medication which is
used for the treatment of extrapyramidal symptoms. Other options are not warranted actions
based on the symptoms presented.

397. One year after being discharged from the burn trauma unit, a client with a history of
40% full-thickness burns is admitted with bone pain and muscle weakness. Which
intervention should the nurse include in the clients plan of care?
A. Encourage Progressive active range of motion
B. Teach need for dietary and supplementary vitamin D3
C. Explain the need for skin exposure to sunlight without sunscreen
D. Instruct the client to use of muscle strengthening exercises
Answer: B. Teach need for dietary and supplementary vitamin D3
Rationale:
Burn injury results in the acute loss of bone as well as the development of progressive
vitamin D deficiency because burn scar tissue and adjacent normal appearing skin cannot
convert normal quantities of the precursors for vitamin D3 that is synthesized from ultraviolet
sun rays which is needed for strong bones. Clients with a history of full thickness burns
should increase their dietary resources of vitamin D and supplemental D3 (B). range of
motion (A) and muscle strengthening exercises (D) do not treat he underlying causes of the
bone pain and weakness unprotected sunlight (C) should be avoided.
398. When teaching a group of school-age children how to reduce the risk of Lyme disease
which instruction should the camp nurse include?
A. Wash hands frequently
B. Avoid drinking lake water
C. Wear long sleeves and pants
D. Do not share personal products
Answer: C. Wear long sleeves and pants
Rationale:
Lyme disease is it tick bone disorder and is transmitted to a child via a tick bite. Keeping the
skin covered reduces the risk of being bitten by a tick. Other options are not reduce the risk
for tick bites.
399. A native-American male client diagnosed with pneumonia, states that in addition to his
prescribed medical treatment of IV antibiotics he wishes to have a spiritual cleaning
performed. Which outcome statement indicates that the best plan of care was followed?
A. Identifies his ethnocentric values and behaviours

B. States an understanding of the medical treatment
C. Participated actively in all treatments regimens
D. Expresses a desire for cultural assimilation
Answer: C. Participated actively in all treatments regimens
Rationale:
Indicates active participation by the client, which is required for treatment to be successful.
The best plan of care should incorporate the valued and treatments of both cultures and in this
case, there is no apparent cultural clash between the two forms of treatment. The client has
already identify he's cultural values (A). (B) Only considers one of the two treatment
modalities desired by the client the client has already chosen how he wishes to assimilate his
cultural values with the prescribed medical treatment (D).
400. A male client with cancer is admired to the oncology unit and tells the nurse that he is in
the hospital for palliative care measures. The nurse notes that the client’s admission
prescription include radiation therapy. What action should the nurse implement?
A. Ask the client about his expected goals for the hospitalization
B. Explain the palliative care measures can be provided at home
C. Notify do radiation department to withhold the treatment for now
D. Determine if the client wishes to cancel further radiation treatment
Answer: A. Ask the client about his expected goals for the hospitalization
Rationale:
Palliative care measures provide relief or control of symptoms, so it is important for the nurse
to determine the client’s goals for symptom control while receiving treatment in the hospital.
Although home care is available the client may not be legible for palliative care at home.
Radiation therapy is an effective positive care measure used to manage symptoms and would
be appropriate unless the radiation conflicts with the client goals.
401. A client with myasthenia Gravis (MG) is receiving immunosuppressive therapy.
Review recent laboratory test results show that the client’s serum magnesium level has
decreased below the normal range. In addition to contacting the healthcare provider, what
nursing action is most important?
A. Check the visual difficulties
B. Note most recent haemoglobin level
C. Assessed for he and Hand joint pain

D. Observe rhythm on telemetry monitor
Answer: D. Observe rhythm on telemetry monitor
Rationale:
If not treated a low little Serum magnesium level can affect myocardial depolarization
leading to a lethal arrhythmia, and the nurse should assess for dysrhythmias before contacting
the healthcare provider. Other choices are common in MG but do not contribute the Safety
risk of low magnesium levels.
402. A young adult female presents at the emergency centre with acute lower abdominal pain.
Which assessment finding is most important for the nurse to report to the healthcare
provider?
A. Pain scale rating at 9 on a 0-10 scale
B. Last menstrual period was 7 weeks ago
C. Reports white curdy vaginal discharge
D. History of irritable bowel syndrome IBS
Answer: B. Last menstrual period was 7 weeks ago
Rationale:
Acute lower abdominal pain in A young adult female can be indicative of an ectopic
pregnancy, which can be life threatening. Since the clients last menstrual period was seven
weeks ago a pregnancy test to be obtained to ruled out ectopic pregnancy, which can result in
intra-abdominal haemorrhage caused by a ruptured Fallopian tube. Although the severity of
pain requires treatment, the most significant finding is the clients last menstrual period. Other
options are not the most important concerns.
403. A 154 pound client with diabetic ketoacidosis is receiving an IV of normal saline 100
ML with regular insulin 100 units. The healthcare provider prescribes a rate of 0.1
units/kg/hour. To deliver the correct dosage, the nurse should set the infusion pump to Infuse
how many ml/hour? enter numeric value only
Answer: 7
Rationale:
Convert the client’s weight to kg, 2.2 pound: 1 kg: 154 pounds: x kg = 154/2.2 = 70kg.
Calculate the client infusion rate, 0.1 x 70 kg = 7 units/hour. Using the formula, D/H x Q = 7
units/hour / 100 units x 100 ml = 7ml / hour

404. The nurse is assessing a postpartum client who is 36 hours post-delivery. Which finding
should the nurse report to the healthcare provider?
A. White blood count of 19,000 mm3
B. Oral temperature of 100.6 F
C. Fundus deviated to the right side
D. Breasts are firm when palpated
Answer: B. Oral temperature of 100.6 F
Rationale:
A temperature greater than 100.4 F (38 C) (B), which is indicative of endometriosis (infection
of the lining of the uterus), should be reported to the health care provider. (A and D) are
findings that are within normal limits in the postpartum period. Fundal deviation to one side
(C) is an expected finding related to a full bladder, so the nurse should encourage the client to
void.
405. A nurse who is working in the emergency department triage area is presented with four
clients at the same time. The client presented with which symptoms requires the most
immediate intervention by the nurse?
A. Low-grade fever, headache, and malaise for the past 72 hours
B. Unable to bear weight on the left foot, with the swelling and bruising
C. Chest discomfort one hour after consuming a large, spicy meal
D. One-inch bleeding laceration on the chain of the crying five-year-old
Answer: C. Chest discomfort one hour after consuming a large, spicy meal
Rationale:
Emergency triage involves quick assessment to prioritize the need for further evaluation and
care. Those with trauma, chest pain, respiratory distress, or acute neurological changes are
priority. In this example, while clients with other conditions require attention, the client with
chest discomfort is at greatest risk and is a priority.
406. The nurse is planning to assess a client's oxygen saturation to determine if additional
oxygen is needed via nasal cannula. The client has a bilateral below-the-knee amputation and
pedal pulses that are weak and threaty. What action should the nurse take?
A. Document that an accurate oxygen saturation reading cannot be obtained
B. Elevate to client's hands for five minutes prior to obtaining a reading from the finger
C. Increase the oxygen based on the clients breathing patterns and lung sounds

D. Place the oximeter clip on the ear lobe to obtain the oxygen saturation reading
Answer: D. Place the oximeter clip on the ear lobe to obtain the oxygen saturation reading
Rationale:
Pulse oximeter clips can be attached to the earlobe to obtain an accurate measurement of
oxygen saturation. Other options will not provide the needed assessment.
407. A young adult male who is being seen at the employee health care clinic for an annual
assessment tell the nurse that his mother was diagnosed with schizophrenia when she was his
age and that life with a schizophrenic mother was difficulty indeed. Which response is best
for the nurse to provide?
A. Ask the client if he is worried about becoming schizophrenic at the age his mother was
diagnosed.
B. Encourage the client to seek genetic counseling to determine his risk for mental illness.
C. Informed the client that his mother schizophrenic has affected his psychological
development.
D. Tell the client that mental illness has a familial predisposition so he should see a
psychiatrist.
Answer: A. Ask the client if he is worried about becoming schizophrenic at the age his
mother was diagnosed.
408. A client on a long-term mental health unit repeatedly takes own pulse regardless of the
circumstance. What action should the nurse implement?
A. Overlook the client’s behavior.
B. Distract client to interfere with the ritual.
C. Ask why the client checks the pulse.
D. Hold client’s hand to stop the behavior.
Answer: A. Overlook the client’s behavior.
409. A client is discharged with automated peritoneal dialysis (PD) to be used nightly…which
instructions should the nurse include?
A. Wash hands before cleaning exit site
B. Keep the head of the bed flat at night
C. Feel for a thrill and a distal pulse nightly
D. Do not get up if fluid is left in the abdomen

Answer: A. Wash hands before cleaning exit site
410. The charge nurse observes the practical nurse (PN) apply sterile gloves in preparation
for performing a sterile dressing change. Which action by the PN requires correction by the
charge nurse?
A. Opening the package
B. Picking up the second glove
C. Picking up the first glove
D. Positioning of the table
Answer: B. Picking up the second glove
411. A male client reports to the clinic nurse that he has been feeling well and is often “dizzy”
his blood pressure is elevated. Based on this findings, this client is at a greatest risk for which
pathophysiological condition?
A. Stroke
B. Renal failure
C. Left ventricular hypertrophy
D. Pulmonary hypertension
Answer: A. Stroke
412. The nurse ask the parent to stay during the examination of a male toddler’s genital area.
Which intervention should the nurse implement?
A. Examine the genitalia as the last part of the total exam.
B. Use soothing statements to facilitate cooperation
C. Allow the child to keep underpants on to examine genitalia
D. Work slowly and methodically so not to stress the child
Answer: A. Examine the genitalia as the last part of the total exam.
413. The nurse is changing a client’s IV tubing and closes the roller clamp on the new tubing
setup when the bag of solution is….which action should the nurse take to ensure adequate
filling of the drip chamber?
A. Lower the IV bag to a flat surface
B. Compress the drip chamber
C. Open the roller clamp

D. Squeeze the bag of IV solution
Answer: B. Compress the drip chamber
414. An Insulin infusion for a client with diabetes mellitus who is experiencing
hyperglycaemic hyperosmolar…in addition to the client’s glucose, which laboratory value is
most important for the nurse to monitor?
A. Serum potassium
B. Urine ketones
C. Urine albumin
D. Serum protein
Answer: A. Serum potassium
415. A young adult who is hit with a baseball bat on the temporal area of the left skull is
conscious when admitted to the ED and is transferred to the Neurological Unit to be
monitored for signs of closed head injury. Which assessment finding is indicative of a
developing epidural hematoma?
A. Altered consciousness within the first 24 hours after injury.
B. Cushing reflex and cerebral edema after 24 hours
C. Fever, nuchal rigidity and opisthotonos within hours
D. Headache and pupillary changes 48 hours after a head injury
Answer: A. Altered consciousness within the first 24 hours after injury.
416. In planning strategies to reduce a client's risk for complications following orthopaedic
surgery, the nurse recognizes which pathology as the underlying cause of osteomyelitis?
A. infectious process
B. metastatic process
C. autoimmune disorder
D. inflammatory disorder
Answer: A. infectious process
417. A client with bipolar disorder began taking valproic acid (Depakote) 250 mg PO three
times daily two months ago. Which finding provides the best indication that the medication
regimen is effective?
Answer: The family reports a great reduction in client’s maniac behavior

418. Which intervention should the nurse implement during the administration of vesicant
chemotherapeutic agent via an IV site in the client’s arm?
Answer: Assess IV site frequently for signs of extravasation
419. A client with a serum sodium level of 125 meq/mL should benefit most from the
administration of which intravenous solution?
A. 0.9% sodium chloride solution (normal saline)
B. 0.45% sodium chloride solution (half normal saline)
C. 10% Dextrose in 0.45% sodium chloride
D. 5% dextrose in 0.2% sodium chloride
Answer: A. 0.9% sodium chloride solution (normal saline)
420. A client with Alzheimer’s disease falls in the bathroom. The nurse notifies the charge
nurse and completes a fall follow-up assessment. What assessment finding warrants
immediate intervention by the nurse?
A. Urinary incontinence
B. Left forearm hematoma
C. Disorientation to surroundings
D. Dislodge intravenous site
Answer: B. Left forearm hematoma
Rationale:
The left forearm hematoma may be indicative an injury, such as broken bone, that requires
immediate intervention. A may be likely be due to the inability to use the toilet due to the fall.
Disorientation is a common symptom of Alzheimer’s disease. IV Dislodged is not an urgent
concern.
421. The nurse is triaging clients in an urgent care clinic. The client with which symptoms
should be referred to the health care provider immediately?
A. headache, photophobia, and nuchal rigidity
B. high fever, skin rash, and a productive cough
C. nausea, vomiting, and poor skin turgor
D. malaise, fever, and stiff, swollen joints
Answer: A. headache, photophobia, and nuchal rigidity

Rationale:
Headache, photophobia, and nuchal rigidity are classic signs of meningeal infection, so this
client should immediately be referred to the health care provider. AC D do not have priority
of B
422. An adult male is brought to the emergency department by ambulance following a
motorcycle accident. He was not wearing a helmet and presents with periorbital bruising and
bloody drainage from both ears. Which assessment finding warrants immediate intervention
by the nurse?
A. Rebound abdominal tenderness
B. nausea and projectile vomit
C. rib pain with deep inspiration
D. diminished bilateral breath sounds
Answer: B. nausea and projectile vomit
Rationale:
Projective vomiting is indicative of increasing intracranial pressure, which can lead to
ischemic brain damage or death, so this finding warrants immediate intervention. Rebound
abdominal tenderness may indicate internal bleeding.
Diminished breath sound may be related to pain. Rib pain with inspiration may indicate rib
fracture.
423. After placement of a left subclavian central venous catheter (CVC), the nurse receives
report of the x-ray findings that indicate the CVC tip is in the client’s superior vena cava.
Which action should the nurse implement?
A. Initiate intravenous fluid as prescribed
B. Notify the HCP of the need to reposition the catheter
C. Remove the catheter and apply direct pressure for 5 minute
D. Secure the catheter using aseptic technique
Answer: A. Initiate intravenous fluid as prescribed
Rationale:
Venous blood return to the heart and drains from the subclavian vein into the superior vena
cava. The X-ray findings indicate proper placement of the CVC, so prescribed intravenous
fluid can be started. A and B are not indicated at this time. The catheter should be secure
immediate following insertion (C)

424. The nurse has received funding to design a health promotion project for African
American women who are at risk for developing breast cancer. Which resource is most
important in designing this program?
A. A listing of African-American women so live in the community
B. Participation of community leaders in planning the program
C. Morbidity data for breast cancer in women of all races
D. Technical assistance to produce a video on breast self-examination.
Answer: B. Participation of community leaders in planning the program
Rationale:
When developing a culturally-competent health promotion project, the participation of
stakeholders and community leaders is most important. A and B might be useful background
information, but t=first the program should be developed. D may be useful fulfilling the plan
developed by the health care team and the community leaders if funding for this assistance is
included in the budget.
425. The home care nurse provide self-care instruction for a client chronic venous
insufficiency cause by deep vein thrombosis. Which instructions should the nurse include in
the client’s discharge teaching plan? Select all that apply
A. Avoid prolonged standing or sitting
B. Use recliner for long period of sitting
C. continue wearing elastic stocking
D. Maintain the bed flat while sleeping
E. Cross legs at knee but not at ankle
Answer: A. Avoid prolonged standing or sitting
B. Use recliner for long period of sitting
C. continue wearing elastic stocking
426. The nurse is interviewing a client with schizophrenia. Which client behavior requires
immediate intervention?
A. Lip smacking and frequent eye blinking
B. Shuffling gait and stooped posture
C. Rocks back and forth in the chair
D. Muscle spasms of the back and neck

Answer: D. Muscle spasms of the back and neck
Rationale:
An extra pyramidal symptom (EPS) characterized by abnormal muscle spasms of the neck
(A) requires immediate intervention because it can cause difficulty swallowing and
jeopardize the airway. Though (A, B and C) are also EPS caused by antipsychotic medication
medications used to manage schizophrenia (D) has the highest priority to insure client safety
is (A)
427. A male client was transferred yesterday from the emergency department to the telemetry
unit because he had ST depression and resolved chest pain. When his EKG monitor alarms
for ventricular tachycardia (VT), what action should the nurse take first?
A. Determine the client’s responsiveness and respirations .
B. Bring the crash cart to the room to defibrillate the client.
C. Immediately initiate chest compressions.
D. Notify the emergency response team
Answer: A. Determine the client’s responsiveness and respirations .
Rationale:
Activities, such as brushing teeth, can mimic the waveform of VI, so first he client should be
assessed (A) to determine if the alarm is accurate. The crash cart can be brought to the room
by someone else and defibrillation (B) delivered as indicated by the client’s rhythm. Based on
as assessment of the client, CPR© as summoning the emergency response team (D) may be
indicated.
428. A client with a large pleural effusion undergoes a thoracentesis. Following the
procedure, which assessment finding warrants immediate intervention by the nurse?
A. The client has asymmetrical chest wall expansion
B. The clients complain of pain at the insertion site
C. The client chest’s x-ray indicates decreased pleural effusion
D. The client’s arterial blood gases are pH 7.35, PaO2 85, Pa CO2 35, HCO3 26
Answer: A. The client has asymmetrical chest wall expansion
Rationale:
A potential complication of thoracentesis is a pneumothorax. The symptoms of a
pneumothorax are uneven, unequal movement of the chest wall. A is an expected finding after

the local anesthetic effects “wear off” B is a desired result of thoracentesis and C is within
normal limits.
429. A client is receiving an IV solution labeled Heparin Sodium 20,000 Units in 5% dextrose
injection 500 ml at 25 ml/hour. How many units of heparin is the client receiving each hour?
Answer: 1000 units/hour
Rationale:
20000/500 = 40 x 25 = 1000
430. The nurse is preparing a client for discharge from the hospital following a liver
transplant. Which instruction is most important for the nurse to include in this client’s
discharge teaching plan?
A. The client has asymmetrical chest wall expansion
B. Measure the abdominal girth daily
C. Report the onset of sclera jaundice
D. Keep a record of daily urinary output
Answer: A. The client has asymmetrical chest wall expansion
Rationale:
The client should be instructed to monitor or elevated temperature because
immunosuppressant agents, which are prescribed to reduce rejection after transplantation,
place the client at risk for infection. The client should recognize sign of liver rejection, such
as sclera jaundice and increasing abdominal girths, but fever may be the only sign of
infection. A is not as important and monitoring for signs of infection.
431. The nurse is conducting health assessments. Which assessment finding increases a
56year-old woman’s risk for developing osteoporosis?
A. Body mass index of (BMI) of 31
B. 20 pack-year history of cigarette smoking
C. Birth control pill usage until age 45
D. Diabetes mellitus in family history
Answer: B. 20 pack-year history of cigarette smoking
Rationale:
Cigarette smoking (2 packs/day x 310 years = 20 packs-year) increases the risk of
osteoporosis. BMI of 30 or greater falls in the category of obesity which increase weight

bearing that is protective against osteoporosis. C contain estrogens, and are also protective
against development of osteoporosis. D is not related to the development of osteoporosis.
432. A young couple who has been unsuccessful in conceiving a child for over a year is seen
in the family planning clinic. During an initial visit, which intervention is most important for
the nurse to implement?
A. Determine current sexual practice
B. Prepare a female client for an ultrasound
C. Request an sperm sample for ovulation
D. Evaluate hormone levels on both client
Answer: A. Determine current sexual practice
Rationale:
First a history should be obtained including practices that might be related to the infertility,
such as douching, daily ejaculation or the male partner’s exposure to heat, such as frequent
sauna or work environment which can decrease sperm production (A B or C) may be
indicated after a complete assessment is obtained.
433. The nurse administers an oral antiviral to a client with shingles. Which finding is most
important for the nurse report to the health care provider?
A. Decreased white blood cell count
B. Pruritus and muscle aches
C. Elevated liver function tests
D. Vomiting and diarrhoea
Answer: C. Elevated liver function tests
Rationale:
Elevated liver function enzymes are a serious side effect of antivirals and should be reported.
A decrease white blood count is a consistent finding with shingle B and (C and D) are side
effects that affect that are of less priority than A.
434. A client in the intensive care unit is being mechanically ventilated, has an indwelling
urinary catheter in place, and is exhibiting signs of restlessness. Which action should the
nurse take first?
A. Review the heart rhythm on cardiac monitors
B. Check urinary catheter for obstruction

C. Auscultated bilateral breath sounds
D. Give PRN dose of lorazepam (Ativan)
Answer: C. Auscultated bilateral breath sounds
Rationale:
Restlessness often results from decreased oxygenation so breath sounds should be assessed
first. Giving an anxiolytic such as lorazepam, might be indicated but first the client should be
assessed for the cause of the restlessness. An obstruction in the urinary drainage system can
cause a distended bladder that may result in restlessness, but patent airway is the priority
intervention. The client should be assessed before evaluating the cardiac rhythm on the
monitor.
435. The nurse makes a supervisory home visit to observe an unlicensed assistive personnel
(UAP) who is providing personal care for a client with Alzheimer’s disease. The nurse
observes that whenever the client gets upset, the UAP changes the subject. What action
should the nurse take in response to this observation?
A. Tell the UAP to offer more choices during the personal care to prevent anxiety
B. Meet with the UAP later to role model more assertive communication techniques
C. Assume care of the client to ensure that effective communication is maintained.
D. Affirm that the UAP is using and effective strategy to reduce the client’s anxiety.
Answer: D. Affirm that the UAP is using and effective strategy to reduce the client’s anxiety.
Rationale:
Reduction is an effective technique is managing the anxiety of client with Alzheimer’s
disease, so the nurse should affirm the UAP is using an effective strategy (A). Nurse assertive
communication and offering more choices (B) may increase… an agitation (C) is not
indicated since the UAP is using redirection, an effective strategy.
436. An older female who ambulate with a quad-cane prefer to use a wheel chair because she
has a halting and unsteady gait at times. Which interventions should the nurse implement?
(Select all that apply)
A. Move personal items within client’s reach
B. Lower bed to the lower possible position
C. Give directions to call for assistance
D. Assist client to the bathroom in 2 hours.
E. Encourage the use of the wheelchair

F. Raise all bed rails when the client is resting
Answer: A. Move personal items within client’s reach
B. Lower bed to the lower possible position
C. Give directions to call for assistance
D. Assist client to the bathroom in 2 hours.
Rationale:
A client who needs assistive devices, such as quad-cane is at risk for falls. Precautions that
should implement include ensuring that personal items are within reach the bed is in the
lowest position and directions are given to call assistance to minimize the risk for falls.
Frequently assisting the client to the bathroom help ensure this client does not go the
bathroom by herself, thereby decreasing the possibility of falling.
437. In evaluating the effectiveness of a postoperative client’s intermittent pneumatic
compression devices, which assessment is most important for the nurse to complete?
A. Evaluate the client’s ability to use an incentive spirometer
B. Monitor the amount of drainage from the client’s incision
C. Observe both lower extremities for redness and swelling
D. Palpate all peripheral pulse points for volume and strength
Answer: C. Observe both lower extremities for redness and swelling
Rationale:
Intermittent compression devices (ICDs) are used to reduce venous stasis and prevent venous
thrombosis in mobile and postoperative clients and its effectiveness is best assessed by
observing the client’s lower extremities for early signs of thrombophlebitis.
438. A school-age child who weighs 42 pounds receives a post-tonsillectomy prescription for
promethazine (Phenergan) 0.5 mg/kg IM to prevent postoperative nausea. The medication is
available in 25 mg/ml ampules. How many ml should the nurse administer? (Enter numeric
value only. If rounding is required, round to the nearest tenth).
Answer: 0.4
Rationale:
Convert pounds to kg 42lbs = 19.09 kg
Next calculate to prescribed dose, 0.5 mg x 1909 kg = 9.545
Then use the desired dose/ dose on hand x volume on hand (9.545/25 x 1ml = 0.3818 = 0.4
ml)

Or use ratio proportion (9.545 mg: x ml = 25 mg: 1ml
25x = 9.545
X = 0.3818 = 0.4)
439. A nurse stops at the site of a motorcycle accident and finds a young adult male lying face
down in the road in a puddle of water. It is raining, no one is available to send for help, and
the cell phone is in the car about 50 feet away. What action should the nurse take first?
A. Examine the victim’s body surfaces for arterial bleeding
B. Stabilize the victim’s neck and roll over to evaluate his status
C. Return to the car to call emergency response 911 for help
D. Open the airway and initiate resuscitative measures
Answer: B. Stabilize the victim’s neck and roll over to evaluate his status
440. During a well-baby, 6-month visit, a mother tells the nurse that her infant has had fewer
ear infections than her 10-year-old daughter. The nurse should explain that which vaccine is
likely to have made the difference in the siblings’ incidence of otitis media?
A. Varicella Virus Vaccine Live
B. Hemophilic Influenza Type B (HiB) vaccine
C. Pneumococcal vaccine
D. Palivizumab vaccine for RSV
Answer: B. Hemophilic Influenza Type B (HiB) vaccine
441. The healthcare provider prescribes Morphine Sulfate Oral Solution 38 mg PO q4 hours
for a client who is opioid-tolerant. The available 30 mL bottle is labelled, 100 mg/5 mL
(20mg/mL), and is packaged with a calibrated oral syringe to provide to provide accurate
dose measurements. How many mL should the nurse administer? (Enter the numerical value
only. If rounding is required, round to the nearest tenth.)
Answer: 1.9
Rationale:
D/H x Q 38/20 x 1 = 1.9 mL
442. The nurses observes that a postoperative client with a continuous bladder irrigation has a
large blood clot in the urinary drainage tubing. What actions should the nurse perform first?
Answer: Observe the amount of urine in the client’s urinary drainage bag

443. Which medication should the nurse anticipate administering to a client who is diagnosed
with myxedema coma?
A. Intravenous administration of thyroid hormones
B. Oral administration of hypnotic agents
C. Intravenous bolus of hydrocortisone
D. Subcutaneous administration of vitamin k
Answer: A. Intravenous administration of thyroid hormones
Rationale:
The high mortality of myxedema coma requires immediate administration of IV thyroid
hormones (A). (B) Is contraindicated, because eves small doses can cause profound
somnolence lasting longer than expected. (C) Is administered to clients diagnosed with
adrenal insufficiency (Addisonian crisis) and (D) to clients who have had an overdose of
warfarin.
444. The nurse who works in labor and delivery is reassigned to the cardiac care unit for the
day because of a low census in labor and delivery. Which assignments is best for the nurse to
give this nurse?
A. Transfer a client to another unit
B. Monitor the central telemetry
C. Perform the admission
D. Assist cardiac nurses with their assignments
Answer: D. Assist cardiac nurses with their assignments
Rationale:
When receiving staff from another specialty unit, the charge nurse should allow the nurse to
assist where possible (D) without taking a client assignment so that the nurse is not asked to
perform unfamiliar skills (A, B, C) are likely to involve skills the nurse is not accustomed to
performing.
445. A client who had an emergency appendectomy is being mechanically ventilated, and soft
wrist restrain are in place to prevent self extubation. Which outcome is most important for the
nurse to include in the client’s plan of care?
A. Understand pain management scale
B. Maintain effective breathing patterns

C. Absence of ventilator associated pneumonia
D. No injuries refer to soft restrains occur
Answer: B. Maintain effective breathing patterns
Rationale:
Basic airway management (B) is the priority. Pain management (A), risk of infection (C), and
prevention of injury (D) do not have the same priority as (C)
446. After a routine physical examination, the healthcare admits a woman with a history of
Systemic Lupus Erythematous (SLE) to the hospital because she has 3+ pitting ankle edema
and blood in her urine. Which assessment finding warrants immediate intervention by the
nurse?
A. Blood pressure 170/98
B. Joint and muscle aches
C. Urine output 300 ml/hr
D. Dark, rust-colored urine
Answer: A. Blood pressure 170/98
447. The nurse is explaining the need to reduce salt intake to a client with primary
hypertension. What explanation should the nurse provide?
A. High salt can damage the lining of the blood vessels
B. Too much salt can cause the kidneys to retain fluid
C. Excessive salt can cause blood vessels to constrict
D. Salt can cause information inside the blood vessels
Answer: B. Too much salt can cause the kidneys to retain fluid
Rationale:
Excessive salt intake can contribute to primary hypertension by causing renal salt retention
which influence water retention that expands blood volume and pressure (ACD) are not
believed to contribute to primary hypertension.
448. In assessing a pressure ulcer on a client’s hip, which action should the nurse include?
A. Determine the degree of elasticity surrounding the lesion
B. Photograph the lesion with a ruler placed next to the lesion
C. Stage the depth of the ulcer using the Braden numeric scale
D. Use a gloved finger to palpate for tunnelling around the lesion

Answer: B. Photograph the lesion with a ruler placed next to the lesion
Rationale:
An ulcer extends into the dermis or subcutaneous tissue and is likely to increase in size and
depth, so assessment should include photograph with measuring device to document the size
of the lesion.
.
449. A nurse is planning discharge care for a male client with metastatic cancer. The client
tells the nurse that he plans to return to work despite pain, fatigue, and impending death.
Which goals is most important to include in this client’s plan of care?
A. Implements decisions about future hospices services within the next 3 months.
B. Marinating pain level below 4 when implementing outpatient pain clinic strategies.
C. Request home health care if independence become compromised for 5 days.
D. Arranges for short term counseling stressors impact work schedule for 2 weeks.
Answer: B. Marinating pain level below 4 when implementing outpatient pain clinic
strategies.
Rationale:
An outpatient pain clinic provides the interdisciplinary services needed to manage chronic
pain. Also the client has a terminal disease and is being discharge home, hospice and health
care are not indicating at this time. Short term counseling is not an option.
450. The first paddle has been placed on the chest of a client who needs defibrillation. Where
should the nurse place the second paddle? (Mark the location where the second paddle should
be placed on the image).
Answer:

451. A client who had an open cholecystectomy two weeks ago comes to the emergency
department with complaints of nausea, abdominal distention, and pain. Which assessment
should the nurse implement?
A. Auscultate all quadrant of the abdomen.
B. Perform a digital rectal exam
C. Palpate the liver and spleen
D. Obtain a hemoccult of the client’s stool
Answer: A. Auscultate all quadrant of the abdomen.
452. The nurse is caring for several clients on a telemetry unit. Which client should the nurse
assess first? The client who is demonstrating
A. A paced rhythm with 100% capture after pacemaker replacement
B. Normal sinus rhythm and complaining of chest pain
C. Atrial fibrillation with congestive heart failure and complaining of fatigue
D. Sinus tachycardia 3 days after a myocardial infarction
Answer: B. Normal sinus rhythm and complaining of chest pain
453. A 12-lead electrocardiogram (ECG) indicates a ST elevations in leads V1 to V4, for a
client who reports having chest pain. The healthcare provider prescribe tissue plasminogen
activator (t-PA). Prior to initiating the infusion, which interventions is most important for the
nurse to implement?
Answer: Complete pre-infusion checklist
454. The nurse is evaluating the health teaching of a female client with condyloma
acuminate. Which statement by the client indicates that teaching has been effective?
A. Early treatment is very effective
B. I will clean my hot tub better
C. These warts are caused by a fungus
D. I need to have regular pap smears
Answer: D. I need to have regular pap smears
455. While the nurse is conducting a daily assessment of an older woman who resides in a
long-term facility, the client begins to cry and tells the nurse that her family has stopped
calling and visiting. What action should the nurse take first?

A. Ask the client when a family member last visited her.
B. Determine the client’s orientation to time and space
C. Review the client’s record regarding social interactions
D. Reassure the client of her family’s love for her
Answer: A. Ask the client when a family member last visited her.
456. A female client with severe renal impairment is receiving enoxaparin (lovenox) 30 mg
SUBQ BID. Which laboratory value due to enoxaparin should the nurse report to the
healthcare provider?
A. creatinine clearance 25 mL/ minute
B. calcium 9 mg/dl
C. hemoglobin 12 grams/dl
D. partial thromboplastin time (PTT) 30 seconds
Answer: A. creatinine clearance 25 mL/ minute
457. The nurse notes an increase in serosanguinous drainage from the abdominal surgical
wound from an obese client. What action should the nurse implement?
Answer: Observe the wound for dehiscence
458. The nurse is assigned to care for clients on a medical unit. Based on the notes taken
during the shift report, which client situation warrants the nurse’s immediate attention?
A. A 10-year-old who is receiving chemotherapy and the infusion pump is beeping
B. A young adult with Crohn’s disease who reports having diarrheal stools
C. An older adult with type 2 diabetes whose breakfast tray arrives 20 minutes late
D. A teenager who reports continued pain 30 minutes after receiving an oral analgesic
Answer: A. A 10-year-old who is receiving chemotherapy and the infusion pump is beeping
Rationale:
An infiltration of a caustic agent can cause tissue damage and children are at greater risk for
fluid volume imbalances
459. A nurse is conducting a physical assessment of a young adult. Which information
provides the best indication of the individual nutritional status?
A. Condition of hair, nails, and skin
B. A 24-hour diet history

C. History of a recent weight loss
D. Status of current petite
Answer: A. Condition of hair, nails, and skin
Rationale:
The assessment of hair, nails, and skin is most effective of long-term nutritional status, which
is important in the healing process.
460. The nurse is preparing to administer an infusion of amino acid-dextrose total parenteral
nutrition (TPN) through a central venous catheter (CVC) line. Which action should the nurse
implement first?
A. Check the TPN solution for cloudiness
B. Attach the IV tubing to the central line
C. Set the infusion pump at the prescribed rate
D. Prime the IV tubing with TPN solution
Answer: A. Check the TPN solution for cloudiness
461. A newly admitted client vomits into an emesis basin as seen in the picture. The nurse
should consult with the healthcare provider before administering which of the client’s
prescribes medications?
A. Clopidogrel (Plavix), an antiplatelet agent, given orally
B. Nitroglycerin (nitro-dur), an antianginal, to be given transdermally
C. Methylprednisolone (solu-medrol), a corticosteroid, to be given IV
D. Furosemide (lasix), a loop diuretic, to be given intravenously
E. Enoxaparin (lovenox), a low-molecular weight heparin to be given subcutaneous
Answer: A. Clopidogrel (Plavix), an antiplatelet agent, given orally
C. Methylprednisolone (solu-medrol), a corticosteroid, to be given IV
E. Enoxaparin (lovenox), a low-molecular weight heparin to be given subcutaneous
462. A client diagnosed with bipolar disorder is going home on a week-end pass. Which
suggestions should give the client’s family to help them prepare for the visit?
A. Encourage the family to plan daily activities to keep the client busy
B. Have friends and family visit the client at a welcome home party
C. Discuss the importance of continuing the usual at-home activities
D. Instruct family to monitor the client’s choice of television programs

Answer: C. Discuss the importance of continuing the usual at-home activities
463. On a busy day, one hour after the shift report is completed, the charge nurse learns that a
female staff nurse who lives one hour away from the hospital forgot her prescription eye
glasses at home. What action should the charge nurse take?
A. Encourage the nurse to purchase reading glasses in the hospital gift shop
B. Request another nurse to assist the staff nurse with her documentation
C. Ask the nurse to return home and get her prescription eyeglasses for work.
D. Tell the staff nurse to take a day off and change her weekly work schedule
Answer: C. Ask the nurse to return home and get her prescription eyeglasses for work.
464. A client with pneumonia has an IV of lactated ringer’s solution infusing at 30ml/hr
current labor.…sodium level of 155 mEq/L, a serum potassium level of 4mEq/L…. what
nursing intervention is most important?
Answer: Obtain a prescription to increase the IV rate
465. After teaching a male client with chronic kidney disease (CKD) about therapeutic
diet…which menu of foods indicates that the teaching was effective? Select all that apply
A. A slice of whole grain toast
B. bowl of cream of wheat
Answer: A. A slice of whole grain toast
B. bowl of cream of wheat
466. When five family members arrive at the hospital, they all begin asking the nurse
questions regarding the prognosis of their critically ill mother. What intervention should the
nurse implement first?
A. Include the family in client’s care
B. Request the chaplain’s presence
C. Ask the family to identify a specific spokesperson
D. age the healthcare provider to speak with family.
Answer: C. Ask the family to identify a specific spokesperson

467. An older male who is admitted for end stage of chronic obstructive pulmonary disease
(COPD) tells the nurse …. The client provides the nurse with a living will and DNR. What
action should the nurse implement?
Answer: Obtain a prescription for DNR
468. A client who is recently diagnosed with type 2 diabetes mellitus (DM) ask the nurse how
this type of diabetes leads to high blood sugar. What Pathophysiology mechanism should the
nurse explain about the occurrence of hyperglycemia in those who have type 2 DM?
Answer: The body cells develop resistance to the action of insulin.
469. During a left femoral artery aortogram, the healthcare provider inserts an arterial sheath
and initiate. Through the sheath to dissolve an occluded artery. Which interventions should
the nurse implement?
A. Instruct the client to keep the left leg straight
B. Observe the insertion site for a hematoma
C. Circle first noted drainage on the dressing
Answer: A. Instruct the client to keep the left leg straight
B. Observe the insertion site for a hematoma
C. Circle first noted drainage on the dressing
470. A client whose wrists are sutured from a recent suicide attempt is been transferred from
a medical unit. Which nursing diagnosis is of the highest priority?
Answer: Risk for self-directed violence related to impulsive actions
471. The nurse reviews the signs of hypoglycemia with the parents of a child with Type I
diabetes mellitus. The parents correctly understand signs of hypoglycemia if they include
which symptoms?
A. Fruity breath odor
B. Polyphagia
C. Diaphoresis
D. Polydipsia
Answer: C. Diaphoresis

472. One day following a total knee replacement, a male client tells the nurse that he is
unable to transfer because it is too painful. What action should the nurse implement?
Answer: Encourage use of analgesics before position change
473. The nurse is caring for a client with hypovolemic shock who is receiving two units of
packed red blood cells (RBCs) through a large bore peripheral IV. What action promotes
maintenance of the client’s cardiopulmonary stability during the blood transfusion?
A. Increase the oxygen flow via nasal cannula if dyspnea is present.
B. Place in a Trendelenburg position to increase cerebral blood flow
C. Monitor capillary glucose measurements hourly during transfusion.
D. Encourage increased intake of oral fluid to improve skin turgor.
Answer: A. Increase the oxygen flow via nasal cannula if dyspnea is present.
474. A client with end-stage liver failure is declared brain dead. The family wants to
discontinue feeding and donate any viable organs. Which action should the nurse take?
Answer: Contact the regional organ procurement agency
475. Which information is more important for the nurse to obtain when determining a client’s
risk for (OSAS)?
A. Body mass index
B. Level of consciousness
C. Self-description of pain
D. Breath sounds
Answer: A. Body mass index
476. During the transfer of a client who had major abdominal surgery this morning, the post
anaesthesia care unit (PACU) nurse reports that the client, who is awake and responsive
continues to report pain and nausea after receiving morphine 2 mg IV and ondansetron 4 mg
IV 45 mints ago. Which elements of SBAR communication are missing from the report given
by the PACU nurse? (Select all that apply)
A. Situation
B. Background
C. Assessment
D. Recommendation

Answer: B. Background
C. Assessment
D. Recommendation
Rationale:
BCD are correct. The current situation is reported regarding the client’s nausea and pain (A).
Based on SBAR communication, critical information about the client’s clinical history (B),
and assessment (C) such as pain scale or vital signs related to client’s response to medication,
are not included, nor are any recommendations for further follow-up (D). (E) Is not a
component of SBAR communication
477. The nurse is triaging victims of a tornado at an emergency shelter. An adult woman who
has been wandering and crying comes to the nurse. What action should the nurse take?
A. Check the client’s temperature, blood sugar, and urine output.
B. Transport the client for laboratory client for laboratory test and electrocardiogram (EKG)
C. Delegate care of the crying client to an unlicensed assistant
D. Send the client to the shelter’s nutrient centre to obtain water and food.
Answer: C. Delegate care of the crying client to an unlicensed assistant
Rationale:
According to the simple triage and Rapid Treatment (START) protocol of triage, the nurse
should determine which client fit the objective of providing the greatest good for the greatest
number of people who are most likely to survive. Delegating the care of the crying person to
an unlicensed assistant allow the nurse to care for the injured who require intervention based
on their ability to breath, maintain circulation and follow simple commands. A and B are not
indicated at this time. Although food and water may be indicative, the woman’s distress
should not be dismissed by sending her to the shelter alone.
478. A client in septic shock has a double lumen central venous catheter with one liter of
0.9% Normal Saline Solution infusing at 1 ml/hour through one lumen and TPN infusing at
50 ml/hr. through one port. The nurse prepared newly prescribed IV antibiotic that should
take 45 mints to infuse. What intervention should the nurse implement?
A. Use a secondary port of the Normal Saline solution to administer the antibiotic.
B. Add the antibiotic to the TPN solution, and continue the normal saline solution.
C. Stop the TPN infusion for the time needed to administer the prescribed antibiotic.
D. Add the antibiotic to the Normal Saline solution and continue both infusions.

Answer: A. Use a secondary port of the Normal Saline solution to administer the antibiotic.
Rationale:
A client in septic shock needs antibiotic administered in a timely manner to ensure
maintenance of therapeutic serum level. The nurse should administer the antibiotic using a
secondary port of the Normal Saline solution. No other medications should be administered
using TPN tubing or solution. TPN not should be place on hold because sudden cessation will
cause rapid change in serum glucose levels. Excessively delays in the administration of the
antibiotics.
479. A male client returns to the mental health clinic for assistance with his anxiety reaction
that is manifested by a rapid heartbeat, sweating, shaking, and nausea while driving over the
bay bridge. What action I the treatment plan should the nurse implement?
A. Tell the client to drive over the bridge until fear is manageable
B. Teach client to listen to music or audio books while driving
C. Encourage client to have spouse drive in stressful places.
D. Recommend that the client avoid driving over the bridge.
Answer: B. Teach client to listen to music or audio books while driving
Rationale:
Desensitization is component in the treatment plan for clients with panic attacks which is best
approached with anxiety-reducing strategies, such as listening to audio book (B) during
situation that precipitate symptoms (A) is a flooding technique that requires professional
guidance.
480. Which intervention should the nurse include in the plan of care for a client with
leukocytosis?
A. Avoid intramuscular injections
B. Monitor temperature regularly
C. Assess skin for petechiae or bruising
D. Implement protective isolation measures
Answer: B. Monitor temperature regularly
481. The nurse is teaching a client about the antiulcer medications ranitidine which was…
statement best describes the action of this drug?
A. It blocks the effects of histamine, causing decreased secretion of acid

B. Ranitidine will neutralize gastric acid and decrease gastric pH
C. This drug provides a protective coating over the gastric mucosa
D. It effectively blocks 97% of the gastric acid secreted in the stomach
Answer: A. It blocks the effects of histamine, causing decreased secretion of acid
482. A client with superficial burns to the face, neck, and hands resulting from a house
fire…which assessment finding indicates to the nurse that the client should be monitored for
carbon monoxide…?
A. Expiratory stridor and nasal flaring
B. Mucous membranes cherry red color
C. Carbonaceous particles in sputum
D. Pulse oximetry reading of 80 percent
Answer: B. Mucous membranes cherry red color
483. A female client who was mechanically ventilated for 7 days is extubated. Two hours
later…productive cough, and her respirations are rapids and shallow. Which intervention is
most important?
A. Review record of recent analgesia
B. Provide frequent pulmonary toilet
C. Prepare the client for intubation
D. Obtain STAT arterial blood gases
Answer: C. Prepare the client for intubation
484. The nurse delegates to an unlicensed assistive personnel (UAP) denture care for a client
with…daily leaving. When making this assignment, which instruction is most important for
the nurse to do?
Answer: Place a washcloth in the sink while cleaning the dentures
485. The nurse is assessing the emotional status of a client with Parkinson’s disease. Which
client finding is most helpful in planning goals to meet the client’s emotional needs?
A. Cries frequently during the interview
B. Stares straight ahead without blinking
C. Face does not convey any emotion
D. Uses a monotone when speaking

Answer: A. Cries frequently during the interview
486. When changing a diaper on a 2-day-old infant, the nurse observes that the baby’s legs
are… this finding, what action should the nurse take next?
A. Notify the healthcare provider
B. Continue care since this is a normal finding
C. Document the finding in the record
D. Perform range of motion to the joint
Answer: A. Notify the healthcare provider
487. A school-aged child was recently diagnosed with celiac disease. Which instruction
should the nurse give the classroom teacher?
Answer: The child should avoid eating homemade cookies and cupcakes during parties
488. The nurse is presenting information about fetal development to a group of parents
with…when discussing cephalocaudal fetal development, which information should the nurse
gives the parents?
A. A set order in fetal development is expected
B. Growth normally occurs within one organ at a time
C. Development progress from head to rump
D. Organ formation is directed by brain development
Answer: C. Development progress from head to rump
489. A client has a prescription for lorazepam 2mg for alcohol withdrawal symptoms. Which
finding… the client?
A. Blood pressure 149/101
B. Irregular pulse rate of 80
C. Oral temperature is 98.9 F (37.1 C)
D. Pain rated 7 on scale 1-10
Answer: A. Blood pressure 149/101
490. A client with end-stage liver failure is declared brain dead. The family wants to
discontinue feeding and donate any viable organs. Which action should the nurse take?
Answer: Contact the regional organ procurement agency

491. A male client who was hit by a car while dodging through traffic is admitted to the
emergency department with intracranial pressure (ICP). A computerized tomography (CT)
scan reveals an intracranial bleed. After evacuation of hematoma, postoperative prescription
include: intubation with controlled mechanical ventilation to PaCO2…what is the
pathophysiological basis for this ventilator settings?
Answer: Hypocapnea reduces ICP
492. During a cardiopulmonary resuscitation of an intubated client, the nurse detects a
palpable pulse throughout the two minutes cycle chest compression and absent breath sounds
over the left lung. What action should the nurse implement?
Answer: Prepare for the endotracheal tube to be repositioned
493. A male client is admitted with burns to his face and neck. Which position should the
nurse place the client to prevent contract?
Answer: Hyperextended with neck supported by a rolled towel.
494. A male client is discharged from the intensive care unit following a myocardial
infarction, and the healthcare provider low-sodium diet. Which lunch selection indicates to
the nurse that this client understands the dietary restrictions?
A. Turkey salad sandwich.
B. Clam chowder
C. Macaroni and cheese
D. Bacon, lettuce, and tomato sandwich
Answer: A. Turkey salad sandwich.
495. The nurse prepares an intravenous solution and tubing for a client with a saline lock, as
seen in the video. Which nurse takes next

Answer: Open the roller clamp on the tubing.
496. The healthcare provider prescribes heparin protocol at18 units/kg/hr for a client with a
possible pulmonary embolism. This client weighs 144 pounds. The available solution is
labeled, heparin sodium 25,000 units in 5% dextrose 250 ml. the nurse should program the
pump to deliver how many ml/hr? (Enter numeric value only. If rounding is require round to
the nearest whole number.)
Answer: 12
Rationale:
144/2.2 = 65kg
18units/kg/hr
65 kg x 18units/kg/hr = 1170 units/hr
25000 units heparin/250 ml of D5W = 100 units heparin per ml of solution
Formula D/H x A = X
497. A client is admitted with a wound on the right hand and associated cellulitis. In assessing
the client’s hand, which finding required most immediate follow-up by the nurse?
A. Cyanotic nailbeds
B. Localized tenderness
C. Diffuse erythema
D. Skin hot to touch
Answer: A. Cyanotic nailbeds
498. The nurse is arranging home care for an older client who has a new colostomy following
a large bowel resection three day. The clients plan to live with a family member. Which
action should the nurse implement? Select all that apply
A. Assess the client for self-care ability
B. Provide pain medication instructions
C. Teach care of ostomy to care provider
Answer: A. Assess the client for self-care ability
B. Provide pain medication instructions
C. Teach care of ostomy to care provider

499. A female client with chronic urinary retention explains double voiding technique to the
nurse by stating she voids partially, hold the remaining urine in her bladder for three minutes,
then voids again to empty her bladder fully. How should the nurse respond?
Answer: Advise the client to empty her bladder fully when she first voids
500. When conducting diet teaching for a client who was diagnosed with
hypoparathyroidism, which foods should the nurse encourage the client to eat?
A. Yogurt.
B. Processed cheese.
C. Nuts
D. Fresh turkey
E. Fresh chicken
Answer: A. Yogurt.
B. Processed cheese.
501. The nurse is assessing a middle-aged adult who is diagnosed with osteoarthritis. Which
factor in this client’s history is a contributor to the osteoarthritis?
A. Long distance runner since high school.
B. Lactose intolerant since childhood
C. Photosensitive to a drug currently taking
D. Recently treated for deep vein thrombosis
Answer: A. Long distance runner since high school.
502. When assessing a male client, the nurse notes that he has unequal lung expansion.
What conclusion regarding this finding is most likely to be accurate? The client has
A. A collapsed lung
B. A history of COPD
C. A chronic lung infection
D. Normally functioning lungs
Answer: A. A collapsed lung
503. The nurse manager is conducting an in-services education program on the fire
evacuation of the newborn recovery. What intervention should the nurse manager disseminate
to the staff?

Answer: Evacuate each infant with mother via wheelchair
504. An adult man reports that he recently experienced an episode of chest pressure and
breathlessness when he was jogging in the neighborhood. He expresses concern because both
of his deceased parents had heart disease and his father was a diabetic. He lives with his male
partner, is a vegetarian, and takes atenolol which maintain his blood pressure at 138/74.
Which risk factors should the nurse explore further with the client? Select all that apply
A. History of hypertension.
B. Family heath history.
Answer: A. History of hypertension.
B. Family heath history.
505. A client with severe full-thickness burns is scheduled for an allografting procedure.
Which information should the nurse provide the client?
Answer: Human source grafts require monitoring for signs of graft rejection
506. The nurse is administering a 750 ml cleansing enema to an adult client. After
approximately150 ml of enema has informed, the client states, ‘stop I can’t hold anymore.”
What action should the nurse take?
Answer: Clamp the tubing and instruct the client to breathe deeply before continuing.
507. The nurse requests a meals tray for a client follows Mormon beliefs and who is on clear
liquid diet following abdominal surgery. Which meal item should the nurse request for this
client? (Select all that apply)
A. Apple juice
B. Chicken broth.
C. Hot chocolate
D. Orange juice
E. Black coffee
Answer: A. Apple juice
B. Chicken broth.

508. The nursing staff on a medical unit includes a registered nurse (RN), practical nurse
(LPN), and unlicensed assistant personnel (UAP). Which task should the charge nurse assign
to the RN?
Answer: Supervised a newly hired graduate nurse during an admission assessment
509. Following breakfast, the nurse is preparing to administer 0900 medications to clients on
a medical floor. Which medication should be held until a later time?
Answer: The mucosal barrier, sucralfate (Carafate), for a client diagnosed with peptic ulcer
disease.
Rationale:
Carafate coats the mucosal lining prior to eating a meal
510. The father of 4-year-old has been battling metastatic lung cancer for the past 2 years.
After discussing the remaining options with his healthcare provider, the client requests that
all treatment stop and that no heroic measures be taken to save his life. When the client is
transferred to the palliative care unit, which action is most important for the nurse working on
the palliative care unit to take in facilitating continuity of care?
Answer: Obtain a detailed report from the nurse transferring the client.
511. The nurse is making a home visit to a male client who is in the moderate stage of
Alzheimer’s diseases. The client’s wife is exhausted and tells the nurse that the family plans
to take turns caring for the client in their home, each keeping him for two weeks at a time.
How should the nurse respond?
Answer: Suggest enrolling the client in adult daycare instead of rotating among family.
512. The healthcare provider prescribes oxycodone/ aspirin 1 tab PO every 4h as needed for
pain, for a client with polycystic kidney disease. Before administering this medication, which
component of the prescription should the nurse question?
A. Aspirin content.
B. Dose
C. Route
D. Risk for addiction
Answer: A. Aspirin content.

513. A young adult male was admitted 36 hours ago for a head injury that occurred as the
result of a motorcycle accident. In the last 4 hours, his urine output has increased to over 200
ml/H. Before reporting the finding to the healthcare provider, which intervention should the
nurse implement?
Answer: Evaluate the urine osmolality and the serum osmolality values.
514. A female client is taking alendronate, a bisphosphate, for postmenopausal osteoporosis.
The client tells the nurse that she is experiencing jaw pain. How should the nurse respond?
Answer: Report the client’s jaw pain to the healthcare provider.
515. A male client has received a prescription for orlistat for weight and nutrition
management. In addition to the medication, the client states he plans to take a multivitamin.
What teaching should the nurse provide?
Answer: Be sure to take the multivitamin and the medication at least two hours apart for best
absorption and effectiveness.
516. Which intervention should the nurse implement for a client with a superficial (first
degree) burn?
Answer: Place wet cloths on the burned areas for short periods of time.
517. What is the primary goal when planning nursing care for a client with degenerative joint
disease (DJD)?
Answer: Achieve satisfactory pain control.
518. An adult woman who is seen in the clinic with possible neuropathic pain of the right leg
rates her pain as a 7 on a 10 point scale. What action should the nurse take?
Answer: Encourage the client to describe the pain.
519. A client has both primary IV infusion and a secondary infusion of medication. An
infusion pump is not available. The nurse needs to determine the current rate of infusion of
the primary IV. Where should the nurse observe to determine the rate of infusion?
Answer:

520. The nurse is conducting the initial assessment of an ill client who is from another
culture…. What response should the nurse provide?
Answer: “What practices do you believe will help you heal?”
521. The nurse is caring a client with NG tube. Which task can the nurse delegate to the
UAP?
Answer: Disconnect the NG suction so the client can ambulate in the hallway
522. The nurse is collecting a sterile urine specimen using a straight catheter tray for
culture…. (Arrange from first action to last).
1. Drape the client in a recumbent position for privacy
2. Open the urinary catheterization tray
3. Don sterile gloves using aseptic technique
4. Use forceps and swaps to clean the urinary meatus
Answer: 1. Drape the client in a recumbent position for privacy
2. Open the urinary catheterization tray
3. Don sterile gloves using aseptic technique
4. Use forceps and swaps to clean the urinary meatus
523. The nurse is caring for a toddler with a severe birth anomaly that is dying. The parents…
holding the child as death approaches. Which intervention is most important for the nurse?
Answer: Notify nursing supervisor and hospital chaplain of the child’s impending death.

524. The nurse is assessing a 4-year-old boy admitted to the hospital with the diagnosis of
possible nephrotic syndrome. Which statement by the parents indicates a likely correlation to
the child’s diagnosis?
Answer: “I couldn’t get my son’s socks and shoes on this morning”
525. Which interventions should the nurse include in a long-term plan of care for a client
with COPD?
A. Reduce risk factors for infection
B. Limit fluid intake to reduce secretions
C. Use diaphragmatic breathing to achieve better exhalation
D. Administer high flow oxygen during sleep
Answer: A. Reduce risk factors for infection
526. A health care provider continuously dismisses the nursing care suggestions made by staff
nurses. As a result…dealing with the healthcare provider. What action should the nurse
manager implement?
Answer: Plan an interdisciplinary staff meeting to develop strategies to enhance client care
527. A 2-year-old girl is brought to the clinic for a routine assessment and all findings are
within the normal limits. However, the mom expresses concern over her daughter’s
protruding abdomen and tells the nurse that she is worry that her child is becoming
overweight. How should the nurse respond to the mother’s comment?
Answer: Explain that a protruding abdomen is typical for toddlers
528. A female client reports she has not had a bowel movement for 3 days, but now is
defecating frequent small amount of liquid stool. Which action should the nurse implement?
Answer: Digitally check the client for a fecal impaction
529. A client admitted with an acute coronary syndrome (ACS) receives eptifibatide, a
glycoprotein (GP) IIB IIIA inhibitor, which important finding places the client at greatest
risk?
Answer: Unresponsive to painful stimuli

530. A toddler presents to the clinic with a barking cough, strider, refractions with respiration,
the child's skin is pink with capillary refill of 2 seconds. Which intervention should the nurse
implement?
Answer: Administered Nebulized Epinephrine
531. The nurse caring for a client with dysphagia is attempting to insert an NG tube, but the
client will not swallow and is not gagging. What action should the nurse implement to
facilitate the NGT passage into the esophagus?
Answer: Flex the client’s head with chin to the chest and insert.
532. The nurse plans to use an electronic digital scale to weight a client who is able to stand.
Which intervention should the nurse implement to ensure that measurement of the client’s
weight is accurate?
Answer: Ensure that the scale is calibrated before a weight is obtained
533. The nurse observes a newly hired unlicensed assistive personnel (UAP) performing a
fingestick to obtain a client’s blood glucose. Prior to sticking the client’s finger, the UAP
explains the procedure and tell the client that it I painless. What action should the nurse take?
Answer: Allow the UAP to complete the procedure, then discuss the painless comment
privately with the UAP.
534. An African-American man come into the hypertension screening booth at a community
fair. The nurse finds that is blood pressure is 170/94 mmHg. The client tells the nurse that he
has never been treated for high blood pressure. What response should the nurse make?
Answer: Your blood pressure is a little high. You need to have it rechecked within one week
535. While attempting to stablish risk reduction strategies in a community, the nurse notes
that the regional studies have indicated….persons with irreversible mental deficiencies due to
hypothyroidism. The nurse should seek funding to implement which screening measure?
Answer: T4 levels in newborns
536. After applying an alcohol-based hand rub to the palms of the hand and rubbing the hand
together, what action should the nurse do next?
Answer: Place one hand on top of the other and interlace the fingers

537. A nurse is preparing to feed a 2-month-old male infant with heart failure who was born
with congenital heart defect. Which intervention should the nurse implement?
Answer: Allow the infant to rest before feeding
538. While removing an IV infusion from the hand of a client who has AIDS, the nurse is
struck with the needle. After washing the puncture site with soap & water, which action
should the nurse take?
Answer: Notify the employee health nurse.
539. A nurse receive a shift report about a male client with Obsessive compulsive disorder
(OCD). The nurse does morning rounds and reaches the client while he is repeatedly washing
the top of the same table. What intervention should the nurse implement?
Answer: Allow time for the behavior and then redirect the clients to other activities
540. The nurse is caring for a client immediately after inserting a PICC line. Suddenly, the
client becomes anxious and tachycardiac, and loud churning is heard over the pericardium
upon auscultation. What action should the nurse take first?
Answer: Place client in Trendelenburg position on the left side.
541. A client admitted to the telemetry unit is having unrelieved chest pain after receiving 3
sublingual nitroglycerin tablets and morphine 8 mg IV. The electrocardiogram reveals sinus
bradycardia with ST elevation. In what order should the nurse implement the nursing actions?
(Arrange first to last)
1. Call the rapid response team to assist
2. Move the crash cart to the client room
3. Notify the client’s healthcare provider
4. Inform the family of the critical situation
Answer: 1. Call the rapid response team to assist
2. Move the crash cart to the client room
3. Notify the client’s healthcare provider
4. Inform the family of the critical situation

542. The nurse is preparing dose # 7 of an IV piggyback infusion of tobramycin for a 73yearol client with... Infected pseudomonas aeruginosa. Which assessment data warrants
further intervention by the nurse?
Answer: Peak and through levels has not been drawn since the tobramycin was started
543. During a left femoral artery aortogram, the healthcare provider inserts an arterial sheath
and initiate…through the sheath to dissolve an occluded artery. Which interventions should
the nurse implement?
A. Instruct the client to keep the left leg straight
B. Observe the insertion site for a hematoma
C. Circle first noted drainage on the dressing
Answer: A. Instruct the client to keep the left leg straight
B. Observe the insertion site for a hematoma
C. Circle first noted drainage on the dressing
544. A client with HIV and pulmonary coccidioidomycosis is receiving amphotericin B.
which assessment finding should the nurse report to the healthcare provider?
Answer: Urinary output of 25mL per hour
545. A client delivers a viable infant, but begins to have excessive uncontrolled
vaginal…notifying the health care provider of the clients’ condition, what information is
most….
A. Maternal blood pressure
B. Maternal apical pulse rate
C. Time Pitocin infusion completed
D. Total amount of Pitocin infused
Answer: A. Maternal blood pressure
546. An infant born to a heroin-addicted mother is admitted to the neonatal care unit. What
behaviours can…to exhibit?
A. Irritability and a high-pitched cry
B. Lethargy and poor suck
C. Facial abnormalities and microcephaly
D. Low birth weight and intrauterine growth retardation

Answer: A. Irritability and a high-pitched cry
547. A multigravida, full-term, labouring client complains of “back labor”. Vaginal
examination reveals that the client’s 3 cm with 50% effacement and the fetal head is at -1
station. What should the nurse implement?
A. Turn the client to a lateral position
B. Apply counter-pressure to the sacral area
C. Notify the scrub nurse to prepare the OR
D. Ambulate the client between contractions
Answer: B. Apply counter-pressure to the sacral area
548. A client with gestational diabetes is undergoing a non-stress test (NST) at 34-week
gestation… is 144 beats/minute. The client is instructed to mark the fetal monitor by pressing
a button each time the baby moves. After 20 minutes, the nurse evaluates the fetal monitor
strip. What…
Answer: Two FHR accelerations of 15 beats/minute x 15 seconds are recorded
549. A male infant born at 28-weeks gestation at an outlying hospital is being prepared for
transport to a respiration are 92 breaths/minute and his heart rate is 156 beats/minute. Which
drug is the transport administration to this infant?
Answer: Instill beractant 100 mg/kg in endotracheal tube.
550. A postpartal client complains that she has the urge to urinate every hour but is only able
to void a small amount. What interventions provides the nurse with the most useful
information?
A. Catheterize for residual urine after next voiding
B. Initiate a perineal pad count
C. Assess for a perineal hematoma
D. Determine the client’s usual voiding pattern
Answer: A. Catheterize for residual urine after next voiding
551. During a 26-week gestation prenatal exam, a client reports occasional dizziness…What
intervention is best for the nurse to recommend to this client?
Answer: Lie on the left or right side when sleeping or resting

552. Artificial rupture of the membrane of a laboring reveals meconium-stained fluid, what
is… the priority?
Answer: Have a meconium aspirator available at delivery
553. A 6-year-old child with acute infectious diarrhoea is placed on a rehydration
therapy…Which action should the nurse instruct the parents to take if the child begins to
vomit?
Answer: Continue giving ORS frequently in small amounts
554. A client is receiving oxytocin (Pitocin) to augment early labor. Which assessment is
most important time the infusion rate is increases?
A. Contraction pattern
B. Blood pressure
C. Infusion site
D. Pain level
Answer: A. Contraction pattern
555. An infant is placed in a radiant warmer immediately after birth. At one hour of age, the
nurse finds the infant tachypneic, and hypotonic. What is the first action that the nurse should
take?
Answer: Determine the infant’s blood sugar level
556. A toddler with a history of an acyanotic heart defect is admitted to the pediatric
intensive…rate of 60 breaths/ minute, and a heart rate of 150 beats/minute. What action
should the nurse take?
Answer: Obtain a pulse oximeter reading
557. In preparing a diabetes education program, which goal should the nurse identify as the
primary emphasis for a class on diabetes self-management?
Answer: Enable clients to become active participants in controlling the disease process
558. To obtain an estimate of a client’s systolic B/P. What action should the nurse take first?
A. Palpate the client’s brachial pulse

B. Pump up the blood pressure cuff
C. Position the stethoscope diaphragm
D. Release the blood pressure cuff valve
Answer: A. Palpate the client’s brachial pulse
559. A client is admitted to isolation with the diagnosis of active tuberculosis (TB). Which
infection control measures should the nurse implement?
A. Negative pressure environment
B. Contact precautions
C. Droplet precautions
D. Protective environment
Answer: A. Negative pressure environment
560. A client is receiving an IV of heparin sodium 25000 units in 5% dextrose injection 500
ml at 14 ml/hour…verify that the client is receiving the prescribed amount of heparin. How
many units is the client receiving?
Answer: 700
Rationale:
25000/500 x 14 = 700
561. A client currently receiving an infusion labelled Heparin Sodium 25,000 Units in 5%
Dextrose Injection 500 mL at 14 mL/hour. A prescription is received to change the rate of the
infusion to 900 units of Heparin per hour. The nurse should set the infusion pump to deliver
how many mL/hour? (Enter numeric value only).
Answer: 18
Rationale:
450000/25000 = 18
562. The nurse notes the client receiving heparin infusion labelled, Heparin Na 25,000 Units
in 5% Dextrose injection 500 ml at 50ml/hr. What dose of Heparin is the client receiving per
hour?
Answer: 2,500

563. A male client is returned to the surgical unit following a left nephrectomy and is
medicated with morphine. His dressing has a small amount of bloody drainage, and a Jackson
Pratt bulb surgical drainage device is in place. Which interventions is most important for the
nurse to include in this clients plan of care?
A. Monitor urine output hourly.
B. Assess for back muscle aches
C. Record drainage from drain
D. Obtain body weight daily
Answer: A. Monitor urine output hourly.
564. The family of a client who just died arrives on the nursing unit after receiving telephone
notification of the death. Several family members state they would like to view the body.
How should the nurse respond?
Answer: Offer to go with the family members to view the body.
565. The nurse is assessing a first day postpartum client. Which finding is most indicative of
a postpartum infection?
Answer: Moderate amount of foul-smelling lochia.
566. An older woman who has difficulty hearing is being discharged from day surgery
following a cataract extraction & lens implantation. Which intervention is most important for
the nurse to implement to ensure the client's compliance with self-care?
Answer: Have the client vocalize the instructions provided.
567. A primigravida client is 36 weeks gestation is admitted to labor and delivery unit
because her membranes ruptured 30minutes ago. Initial assessment indicates 2cm dilation,
50% effaced, -2 station, vertex presentation greenish coloured amniotic fluid, and
contractions occurring 3-5 minutes with a low FHR after the last contraction peaks:
A. Administer Oxygen via face mask
B. Apply an internal fetal heart monitor
C. Notify the healthcare provider
D. Use a vibroacoustic stimulator
Answer: A. Administer Oxygen via face mask

568. A woman just received the Rubella vaccine after a delivery of a normal new born, has
two children at home, ages 13 months and 3 years. Which instruction is most important to
provide to the client?
Answer: Do not get pregnant for at least 3 months
569. Following a motor vehicle collision (MCV), a male adult in severe pain is brought to the
emergency department via ambulance. His injured left leg is edematous, ecchymotic around
the impact of injury on the thigh, and shorter than his right leg. Based on these findings, the
client is at greatest risk for which complication?
A. Arterial ischemia
B. Tissue necrosis
C. Fat embolism
D. Nerve damage
Answer: C. Fat embolism
570. A 2-year-old is bleeding from a laceration on the right lower extremity that occurred as
the result of a motor vehicle collision. The nurse is selecting supplies to start an IV access.
Which assessment finding is most significant in the nurse's selection of catheter size?
A. Thready brachial pulse.
B. Respirations of 24/minute
C. Right foot cool to touch
D. Swelling at the site of injury
Answer: A. Thready brachial pulse.
571. The nurse prepares to insert an oral airway by first measuring for the correct sized
airway. Which picture shows the correct approach to airway size measurement?
Answer:

572. A client with a recent colostomy expresses concern about the ability to control flatus.
Which intervention is most important for the nurse to include in the client’s plan of care?
A. Adhere to a bland diet whenever planning to eat out
B. Decrease fluid intake at meal times

C. Avoid foods that caused gas before the colostomy
D. Eliminate foods high in cellulose
Answer: C. Avoid foods that caused gas before the colostomy
573. A male client arrives at the clinic with a severe sunburn and explains that he did not use
sun screen because it was an overcast day. Large blisters are noted over his back and chest
and his shirt is soaked with serosanguinous fluid. Which assessment finding warrants
immediate intervention by the nurse?
A. Hypotension.
B. Fever and chills
C. Dizziness
D. Headache
Answer: A. Hypotension.
574. A client with polycystic kidney disease (PKD) receiving antibiotics for an infected cyst
is experiencing severe pain. What action should the nurse implement?
A. Hold the next dose of antibiotic until contacting the healthcare provider
B. Teach the client how to use a dry heating pad over the painful area
C. Encourage the client to practice pelvic floor exercises every hour
D. Assist the client to splint the site by applying an abdominal binder
Answer: B. Teach the client how to use a dry heating pad over the painful area
575. Which statement is accurate regarding the pathological changes in the pulmonary system
associated with acute (adult) respiratory distress syndrome (ARDS)?
A. Capillary hydrostatic pressure exceeds colloid osmotic pressure, producing interstitial
edema
B. A high ventilation-to-perfusion ratio is characteristic of affected lung fields in ARDS
C. Functional residual capacity and lung compliance increase as the disease progresses
D. Interstitial edema that occurs due to capillary fluid shifts is usually more serious than
alveolar edema
Answer: A. Capillary hydrostatic pressure exceeds colloid osmotic pressure, producing
interstitial edema

576. The nurse mixes 250 mg of debutamine in 250 ml of D5W and plans to administer the
solution at rate client weighing 110 pounds. The nurse should set the infusion pump to
administer how many ml per hour only. If rounding is required, round the nearest whole
number.)
Answer: 45
577. During the intraoperative phase of care, the circulating nurse observes that the client is
not adequately client's privacy. What is the best initial nursing action for the nurse to
implement?
Answer: Instruct the scrub nurse to re-drape the client
578. An adult male who was admitted two days ago following a cerebrovascular accident
(CVA) is confused and experiencing left-side weakness. He has tried to get out of bed several
times, but is unable to ambulate without assistance. Which intervention is most important for
the nurse to implement?
A. Ask a family member to sit with the client
B. Apply bilateral soft wrist restraints
C. Assign staff to check client q15 minutes
D. Install a bed exit safety monitoring device
Answer: D. Install a bed exit safety monitoring device
579. A client in her first trimester of pregnancy complains of nausea. Which complementary
therapy should the nurse recommend?
A. Drink chamomile tea at breakfast and in the evening.
B. Eat food high in garlic with the evening meal
C. Join a yoga class that meets at least weekly
D. Increase cocoa in the diet and drink before bedtime
Answer: A. Drink chamomile tea at breakfast and in the evening.
580. When gathering for a group therapy session at 1400 hours, a female client complains to
the nurse that a smoking break has not been allowed all day. The nurse responds that 15
minute breaks were called over the unit intercom after breakfast and after lunch. The nurse is
using what communication technique in responding to the client?
A. Doubt

B. Observation
C. Confrontation
D. Reflection
Answer: D. Reflection
581. A female client with rheumatoid arthritis (RA) comes to the clinic complaining of joint
pain and swelling. The client has been taking prednisone (Deltasone) and ibuprofen (Motrin
Extra Strength) every day. To assist the client with self-management of her pain, which
information should the nurse obtain?
A. Presence of bruising, weakness, or fatigue
B. Therapeutic exercise included in daily routine.
C. Average amount of protein eaten daily
D. Existence of gastrointestinal discomfort
Answer: B. Therapeutic exercise included in daily routine.
582. The charge nurse of the Intensive Care Unit is making assignments for the permanent
staff and one RN who was floated from a medical unit. The client with which condition is the
best to assign to the float nurse?
A. Diabetic ketoacidosis and titrated IV insulin infusion
B. Emphysema extubated 3 hours ago receiving heated mist
C. Subdural hematoma with an intracranial monitoring device
D. Acute coronary syndrome treated with vasopressors
Answer: A. Diabetic ketoacidosis and titrated IV insulin infusion
583. A client admitted to the emergency center had inspiratory and expiratory wheezing, nasal
flaring, and thick, tenacious sputum secretions observed during the physical examination.
Based on these assessment findings, what classification of pharmacologic agents should the
nurse anticipate administering?
A. Beta blockers
B. Bronchodilators
C. Corticosteroids
D. Beta-adrenergics
Answer: B. Bronchodilators

584. The home health nurse is assessing a male client who has started peritoneal dialysis (PD)
5 days ago. Which assessment finding warrants immediate intervention by the nurse?
A. Finger stick blood glucose 120 mg/dL post exchange
B. Arteriovenous (AV) graft surgical site pulsations.
C. Anorexia and poor intake of adequate dietary protein
D. Cloudy dialysate output and rebound abdominal pain
Answer: D. Cloudy dialysate output and rebound abdominal pain
585. A client’s telemetry monitor indicates ventricular fibrillation (VF). What should the
nurse do first?
A. Administer epinephrine IV
B. Give an IV bolus of amiodarone
C. Provide immediate defibrillation
D. Prepare for synchronized cardioversion
Answer: C. Provide immediate defibrillation
586. In conducting a health assessment, the nurse determines that both parents of a child with
asthma smoke cigarettes. What recommendation is best to the nurse to recommend to the
parents?
A. avoid smoking in the house
B. stop smoking immediately
C. decrease the number of cigarettes smoke daily
D. obtain nicotine patches to assist in smoking sensation
Answer: A. avoid smoking in the house
587. A client who is schedule for an elective inguinal hernia repair today in day surgery is
seem eating in the waiting area. What action should be taken by the nurse who is preparing to
administer the preoperative medications?
A. Review the surgical consent with the client
B. Explain that vomiting can occur during surgery
C. Remove the food from the client
D. Withhold the preoperative medication
Answer: B. Explain that vomiting can occur during surgery
D. Withhold the preoperative medication

588. The nurse is developing a plan of care for a middle-aged woman who is diagnosed with
type 2 diabetes mellitus (DM). To lower her blood glucose and increase her serum highdensity lipoprotein (HDL) levels, which instruction is most important for the nurse to
provide?
A. Exercise at least three times weekly
B. Monitor blood glucose levels daily
C. Limit intake of foods high in saturated fat
D. Learn to read all food product labels
Answer: A. Exercise at least three times weekly
589. A client who has been in active labor for 12 hours suddenly tells the nurse that she has a
strong urge to have a bowel movement. What action should the nurse take?
A. Allow the client to use a bedpan.
B. Assist the client to the bathroom
C. Perform a sterile vaginal exam
D. Explain the fetal head is descending.
Answer: C. Perform a sterile vaginal exam
Rationale:
When a client in active labor suddenly expresses the urge to have a bowel movement, a sterile
vaginal exam should be performed to determine if the fetus is descending.
590. The nurse assesses a 78-year-old male client who has left sides heart failure. Which
symptoms would the nurse expect this client to exhibit?
A. Dyspnea, cough, and fatigue.
B. Hepatomegaly and distended neck veins
C. Pain over the pericardium and friction rub.
D. Narrowing pulse pressure and distant heart sounds.
Answer: A. Dyspnea, cough, and fatigue.
591. A female client comes to the clinic complaining of fatigue and inability to sleep because
she is the full-time caretaker for 22-year-old son who was paralyzed by a motor vehicle
collision. She adds that her husband left her because he says he can’t take her behavior any
more since all she does is care for their son. What intervention should the nurse implement?

A. Schedule a home visit in the afternoon to assess the son and client role as caregiver.
B. Acknowledge the client’s stress and suggest that she consider respite care.
C. Provide feedback to the client about her atonement for guilt about her son’s impairment.
D. Teach the client to problem-solve for herself and establish her own priorities.
Answer: B. Acknowledge the client’s stress and suggest that she consider respite care.
Rationale:
When this amount of disclosure is offered, the client is usually seeking information focuses
on the client’s expression of worry, concern and stress and addresses the client’s need to
initiate a request for assistance with respite care.
592. The nurse plans to administer a schedule dose of metoprolol (Toprol SR) at 0900 to a
client with hypertension. At 0800, the nurse notes that client’s telemetry pattern shows a
second degree heart block with a ventricular rate of 50. What action should the nurse take?
A. Administer the Tropol immediately and monitor the client until the heart rate increases.
B. Provide the dose of Tropol as scheduled and assign a UAP to monitor the client’s BP q30
minutes.
C. Give the Tropol as scheduled if the client’s systolic blood pressure reading is greater than
180.
D. Hold the scheduled dose of Tropol and notify the healthcare provider of the telemetry
pattern.
Answer: D. Hold the scheduled dose of Tropol and notify the healthcare provider of the
telemetry pattern.
Rationale:
Beta blockers such as metoprolol (Tropol SR) are contraindicated in clients with second or
third degree heart block because they decrease the heart rate. Therefore, the nurse should hold
the medication.
593. A client who developed syndrome of inappropriate antidiuretic hormone (SIADH)
associated with small carcinoma of the lung is preparing for discharge. When teaching the
client about self-management with demeclocycline (Declomycin), the nurse should instruct
the client to report which condition to the health care provider?
A. Insomnia
B. Muscle cramping
C. Increase appetite

D. Anxiety.
Answer: B. Muscle cramping
Rationale:
SIADH causes dilution hyponatremia because of the increased release of ADH, which is
treated with water restriction and demeclocycline, a tetracycline derivate that blocks the
action of ADH. Signs of hyponatremia (normal 136-145), which indicate the need for
increasing the dosage of demeclocycline, should be reported to the healthcare provider. The
signs include: plasma sodium level less than 120, anorexia, nausea, weight changes related to
fluid disturbance, headache, weakness, fatigue, and muscle cramping. AC& D are not related
to hyponatremia.
594. In determine the client position for insertion of an indwelling urinary catheter, it is most
important for the nurse to recognize which client condition?
A. High urinary PH
B. Abdominal Ascites
C. Orthopnea
D. Fever.
Answer: C. Orthopnea
Rationale:
If the client is orthopneic, the nurse needs to adapt the insertion position that does not place
the client in a supine position (the head of the bed should be elevated as much as possible).
595. The nurse is reviewing a client’s electrocardiogram and determines the PR interval (PRI)
is prolonged. What does this finding indicate?
A. Initiation of the impulses from a location outside the SA node
B. Inability of the SA node to initiate an impulse at the normal rate
C. Increased conduction time from the SA node to the AV junction
D. Interference with the conduction through one or both ventricles.
Answer: B. Inability of the SA node to initiate an impulse at the normal rate
Rationale:
A prolonged PRI reflects an increased amount of time for an impulse to travel from the SA
node through the AV node and is characteristic of a first degree heart block.

596. The nurse is teaching a male client with multiple sclerosis how to empty his bladder
using the Crede Method. When performing a return demonstration, the client applies pressure
to the umbilical areas of his abdomen. What instruction should the nurse provide?
A. Stroke the inner thigh below the perineum to initiate urinary flow
B. Contract, hold, and then relax the pubococcygeal muscle
C. Pour warm water over the external sphincter at the distal glans
D. Apply downward manual pressure at the suprapubic regions.
Answer: D. Apply downward manual pressure at the suprapubic regions.
Rationale:
The Crede Method is used for those clients with atonic bladders, which is a concomitant of
demyelinating disorders like multiple sclerosis. The client is applying pressure in the wrong
region (umbilical Are) and should be instructed to apply pressure at the suprapubic are.
597. A 35 years old female client has just been admitted to the post anesthesia recovery unit
following a partial thyroidectomy. Which statement reflects the nurse’s accurate
understanding of the expected outcome for the client following this surgery?
A. Supplemental hormonal therapy will probably be unnecessary
B. The thyroid will regenerate to a normal size within a few years.
C. The client will be restricted from eating seafood
D. The remainder of the thyroid will be removed at a later date.
Answer: C. The client will be restricted from eating seafood
598. A client with gestational diabetes, at 39 weeks of gestation, is in the second stage of
labor. After delivering of the fetal head, the nurse recognizes that shoulder dystocia is
occurring. What intervention should the nurse implement first?
A. Prepare the client for an emergency cesarean birth
B. Encourage the client to move to a hands-and-knees position.
C. Assist the client to sharply flex her thighs up again the abdomen.
D. Lower the head of the bed an apply suprapubic pressure.
Answer: C. Assist the client to sharply flex her thighs up again the abdomen.
Rationale:
Flexing the client’s thighs against the abdomen (Mc Robert’s maneuver) changes the angle o
the pelvis and increase the pelvic diameter, making more room for the shoulders to emerge.
ABD are implemented after C

599. The nurse should observe most closely for drug toxicity when a client receives a
medication that has which characteristic?
A. Low bioavailability
B. Rapid onset of action
C. Short half life
D. Narrow therapeutic index.
Answer: D. Narrow therapeutic index.
Rationale:
A drug with a narrow therapeutic index has a high risk for toxicity because there is a narrow
range between the therapeutic dose and the toxic dose.
600. Following insertion of a LeVeen shunt in a client with cirrhosis of the liver, which
assessment finding indicates to the nurse that the shunt is effective?
A. Decrease abdominal girth
B. Increased blood pressure
C. Clear breath sounds
D. Decrease serum albumin.
Answer: A. Decrease abdominal girth
601. When finding a client sitting on the floor, the nurse calls for help from the unlicensed
assistive personnel (UAP). Which task should the nurse ask the UAP to do?
A. Check for any abrasions or bruises.
B. Help the client to stand.
C. Get a blood pressure cuff.
D. Report the fall to the nurse-manager.
Answer: C. Get a blood pressure cuff.
602. During the initial newborn assessment, the nurse finds that a newborn's heart rate is
irregular. Which intervention should the nurse implement?
A. Notify the pediatrician immediately.
B. Teach the parents about congenital heart defects.
C. Document the finding in the infant's record.
D. Apply oxygen per nasal cannula at 3 L/min.

Answer: C. Document the finding in the infant's record.
603. Which assessment finding indicates to the nurse a client’s readiness for pulmonary
function tests?
Answer: Expresses an understanding of the procedure.
604. A young adult male is admitted to the emergency department with diabetic ketoacidosis
(DKA). His pH is 7.25, HCO3 is 12 mEq/L or 12 mmol/L (SI), and blood glucose is 310
mg/dl or 17.2 mmol/L (SI). Which action should the nurse implement?
Answer: Infuse sodium chloride 0.9% (normal saline)
605. The nurse is assessing the thorax and lungs of a client who is having respiratory
difficulty. Which finding is most indicative of respiratory distress?
Answer: Contractions of the sternocleidomastoid muscle
606. After receiving lactulose, a client with hepatic encephalopathy has several loose stools.
What action should the nurse implement?
Answer: Monitor mental status.
607. A client present at the clinic with blepharitis. What instructions should the nurse provide
for home care?
Answer: Apply warm moist compresses then gently scrub eyelids with dilute baby shampoo
608. Dopamine protocol is prescribed for a male client who weigh 198 pounds to maintain
the mean arterial pressure (MAP) greater than 65 mmHg. His current MAP is 50 mmHg, so
the nurse increases the infusion to 7 mcg/kg/minute. The infusion is labeled dextrose 5% in
water (D5W) 500 ml with dopamine 400 mg. The nurse should program the infusion pump to
deliver how many ml/hour?
Answer: 47
609. The nurse is teaching a client with atrial fibrillation about a newly prescribed
medication, dronedarone. Which information should the nurse include in client interactions?
(Select all that apply)
A. Avoid eating grapefruit or drinking grapefruit juice.

B. Report changes in the use of daily supplements
C. Notify you heal care provider if your skin looks yellow
Answer: A. Avoid eating grapefruit or drinking grapefruit juice.
B. Report changes in the use of daily supplements
C. Notify you heal care provider if your skin looks yellow
610. A male client recently released from a correctional facility arrives at the clinic with a
cough, fever, and chills. His history reveals active tuberculosis (TB) 10 years ago. What
action should the nurse implement? (Select all that apply)
A. Schedule the client for the chest radiograph
B. Obtain sputum for acid fast bacillus (AFB) testing
C. Place a mask on the client until he is moved to isolation.
Answer: A. Schedule the client for the chest radiograph
B. Obtain sputum for acid fast bacillus (AFB) testing
C. Place a mask on the client until he is moved to isolation.
611. A 16-year-old male is admitted to the pediatric intensive care unit after being involved in
a house fire. He has full thickness burns to his lower torso and extremities. Before a dressing
change to his legs, which intervention is most important for the nurse to implement?
Answer: Maintain strict aseptic technique.
612. While performing a skin inspection for a female adult client, the nurse observes a rash
that is well circumscribed, has silvery scales and plaques, and is located on the elbows and
knees. These assessment findings are likely to indicate which condition?
A. Tinea corporis
B. Herpes zoster
C. Psoriasis
D. Drug reaction
Answer: C. Psoriasis
613. A client with acute pancreatitis is complaining of pain and nausea. Which interventions
should the nurse implement (Select all that apply)
A. Monitor heart, lung, and kidney function.
B. Notify healthcare provider of serum amylase and lipase levels.

C. Review client’s abdominal ultrasound findings.
D. Position client on abdomen to provide organ stability
E. Encourage an increased intake of clear oral fluids
Answer: A. Monitor heart, lung, and kidney function.
B. Notify healthcare provider of serum amylase and lipase levels.
C. Review client’s abdominal ultrasound findings.
614. A nurse is caring for a client with Diabetes Insipidus. Which assessment finding
warrants immediate intervention by the nurse?
A. Hypernatremia
B. Excessive thirst
C. Elevated heart rate
D. Poor skin turgor
Answer: A. Hypernatremia
615. In caring for a client receiving the amino glycoside antibiotic gentamicin, it is most
important for the nurse to monitor which diagnostic test?
Answer: Serum creatinine
616. The nurse weighs a 6-month-old infant during a well-baby check-up and determines that
the baby’s weight has tripled compared to the birth weight of 7 pounds 8 ounces. The mother
asks if the baby is gaining enough weight. What response should the nurse offer?
A. What food does your baby usually eat in a normal day?
B. What was the baby’s weight at the last well-baby clinic visit?
C. The baby is below the normal percentile for weight gain
D. Your baby is gaining weight right on schedule
Answer: A. What food does your baby usually eat in a normal day?
617. A client who is at 36 weeks gestations is admitted with severe preclampsia. After a 6
gram loading dose of magnesium sulfate is administered, an intravenous infusion of
magnesium sulfate at a rate of 2 grams/hour is initiated. Which assessment finding warrants
immediate intervention by the nurse?
Answer: Urine output 20 ml/hour

618. What is the nurse’s priority goal when providing care for a 2-year-old child
experience…
A. Stop the seizure activity
B. Decrease the temperature
C. Manage the airway
D. Protect the body from injury
Answer: C. Manage the airway
619. The nurse is preparing to discharge an older adult female client who is at risk for
hy…nurse include with this client’s discharge teaching?
A. Report any muscle twitching or seizures
B. Take vitamin D with calcium daily
C. Low fat yogurt is a good source of calcium
D. Keep a diet record to monitor calcium intake
E. Avoid seafood, particularly selfish
Answer: A. Report any muscle twitching or seizures
B. Take vitamin D with calcium daily
C. Low fat yogurt is a good source of calcium
D. Keep a diet record to monitor calcium intake
620. The husband of a client with advanced ovarian cancer wants his wife to have every
treatment available. When the husband leaves, the client tells the nurse that she has had
enough chemotherapy and wants to stop all treatments but knows her husband will sign the
consent form for more treatment. The nurse’s response should include which information?
A. The husband cannot sign the consent for the client, her signature is required
B. The client’s specific wishes should be discussed with her healthcare provider
C. The healthcare team will formulate a plan of care to keep the client comfortable
Answer: A. The husband cannot sign the consent for the client, her signature is required
B. The client’s specific wishes should be discussed with her healthcare provider
C. The healthcare team will formulate a plan of care to keep the client comfortable
621. The nurse is preparing a 50 ml dose of 50% dextrose IV for a client with
insulin…medication?
Answer: Push the undiluted Dextrose slowly through the currently infusion IV

622. The daughter of an older female client tells the clinic nurse that she is no longer able to
care for her mother since her mother has lost the ability to perform activities of daily living
(ADLs) due to aging. Which options should the nurse discuss with the daughter?
A. Home hospice agency
B. Long-term care facility
C. Rehabilitation facility
D. Independent senior apartment
E. Home health agency
Answer: E. Home health agency
623. A male client with cancer, who is receiving antineoplastic drugs, is admitted to
the…what findings is most often manifest this condition?
A. Ecchymosis and hematemesis
B. Weight loss and alopecia
C. Weakness and activity intolerance
D. Sore throat and fever
Answer: A. Ecchymosis and hematemesis
624. A 7-year-old boy is brought to the clinic because of facial edema. He reports that he has
been voiding small amounts of dark, cloudy, tea-colored urine. The parents state that their son
had a sore throat 2 weeks earlier, but it has resolved. After assessing the child’s vital signs
and weight, what intervention should the nurse implement next?
A. Measure the child’s abdominal girth
B. Perform an ostoscopic examination
C. Collect a urine specimen for routine urinalysis
D. Obtain a blood specimen for serum electrolytes
Answer: C. Collect a urine specimen for routine urinalysis
625. The nurse observes an adolescent client prepare to administer a prescribed corticosteroid
medication using a metered dose inhaler as seen in the picture. What action should the nurse
take?
A. Remind the client to hold his breath after inhaling the medication
B. Confirm that the client has correctly shaken the inhaler

C. Affirm that the client has correctly positioned the inhaler
D. Ask the client if he has a spacer to use for this medication
Answer: A. Remind the client to hold his breath after inhaling the medication


626. The nurse teaches an adolescent male client how to use a metered dose inhaler. Seen in
the picture. What instruction should the nurse provide?
A. Move the device one to two inches away from the mouth
B. Secure the mouthpiece under the tongue
C. Press down on the device after breathing in fully
D. Breathe out slowly and deeply while compressing the device
Answer: A. Move the device one to two inches away from the mouth
627. A 3-year-old boy with a congenital heart defect is brought to the clinic by his
mother…During the assessment, the mother asks the nurse why her child is at the 5th
percent…response is best for the nurse to provide?
A. Does your child seem mentally slower than his peers also?
B. “His smaller size is probably due to the heart disease”
C. Haven’t you been feeding him according to recommended daily allowances for children?
D. You should not worry about the growth tables. They are only averages for children
Answer: B. “His smaller size is probably due to the heart disease”
628. A client with hypertension receives a prescription for enalapril, an
angiotensin…instruction should the nurse include in the medication teaching plan?
A. Increase intake of potassium-rich foods
B. Report increased bruising of bleeding
C. Stop medication if a cough develops

D. Limit intake of leafy green vegetables
Answer: B. Report increased bruising of bleeding
629. When administering ceftriaxone sodium (Rocephin) intravenously to a client
before…most immediate intervention by the nurse?
A. Stridor
B. Nausea
C. Headache
D. Pruritis
Answer: A. Stridor
630. The nurse is assessing a client with a small bowel obstruction who was hospitalized 24
hours ago. Which assessment finding should the nurse report immediately to the healthcare
provider?
A. Rebound tenderness in the upper quadrants
B. Hypoactive bowel sounds in the lower quadrants
C. Tympany with percussion of the abdomen
D. Light colored gastric aspirate via the nasogastric tube
Answer: A. Rebound tenderness in the upper quadrants
631. An adult female client is admitted to the psychiatric unit because of a complex
handwashing ritual she performs daily that takes two hours or longer to complete. She
worries about staying clean and refuses to sit on any of the chairs in the day area. This
client’s handwashing is an example of which clinical behavior?
A. Addiction
B. Phobia
C. Compulsion
D. Obsession
Answer: C. Compulsion
632. A female client reports that she drank a liter of a solution to cleanse her
intestines…immediately. How many ml of fluid intake should the nurse document? Whole
number
Answer: 760

Rationale:
1L = 1000ml
Subtract the emesis, 1 cup (8 oz) = 240ml
1000-240 = 760 ml
633. Following routine diagnostic test, a client who is symptom-free is diagnosed with
Paget’s disease. Client teaching should be directed toward what important goal for this client?
A. Maintain adequate cardiac output
B. Promote adequate tissue perfusion
C. Promote rest and sleep
D. Reduce the risk for injury
Answer: D. Reduce the risk for injury
634. The mother of a one-month-old boy born at home brings the infant to his first well…was
born two weeks after his due date, and that he is a “good, quiet baby” who
almost…hypothyroidism, what question is most important for the nurse to ask the mother?
Answer: Is your son sleepy and difficult to feed?
635. In preparing assignments for the shift, which client is best for the charge nurse to assign
to a practical nurse (PN)?
Answer: An older client post-stroke who is aphasic with right-sided hemiplegia
636. Following a gun shot wound to the abdomen, a young adult male had an emergency
bowel…Multiple blood products while in the operating room. His current blood pressure is
78/52…He is being mechanically ventilated, and his oxygen saturation is 87%. His laboratory
values…Grams / dl (70 mmol / L SI), platelets 20,000 / mm3 (20 x 10 9 / L (SI units), and
white blood cells. Based on these assessments findings, which intervention, should the nurse
implements first?
Answer: Transfuse packed red blood cells
637. After checking the fingerstick glucose at 1630, what action should the nurse implement?
Answer: Administer 8 units of insulin aspart SubQ

638. Progressive kyphoscoliosis leading to respiratory distress is evident in a client with
muscle…Which finding warrants immediate intervention by the nurse?
Answer: Evidence of hypoventilation
639. An adult male who lives alone is brought to the Emergency Department by his daughter
who is unresponsive. Initial assessment indicated that the client has minimal respiratory
effort, and his pupils are fixed and dilated. At the daughter’s request, the client is intubated
and…Which nursing intervention has the highest priority?
Answer: Determine if the client has an executed living will
640. The nurse determines that a client’s pupils constricts as they change focus from a far
object. What documentation should the nurse enter about this finding?
Answer: Pupils reactive to accommodation
641. Which nursing intervention has the highest priority for a multigravida who delivered…
Answer: Assess fundal tone and lochia flow
642. A client who had a gestational trophoblastic disease (GTD) evacuated 2 days ago is
being…18 months-old child and lives in a rural area. Her husband takes the family car to
work daily…transportation during the day. What intervention is most important for the nurse
to implement?
Answer: Schedule a weekly home visit to draw hCG values.
643. A newly graduated female staff nurse approaches the nurse manager and request
reassignment to another client because a male client is asking her for a date and making
suggestive comments. Which response is best for the nurse manager to provide?
Answer: I’ll change your assignment, but let’s talk about you a nurse should respond to this
kind of client.
644. After removing a left femoral arterial sheath, which assessment finding warrant
immediately interventions by the nurse? (Select all that applied.)
A. Unrelieved back and flank pain.
B. Quarter-size red drainage at site
C. Cool and pale left leg and foot.

D. Tenderness over insertion site
E. Left groin egg-size hematoma.
Answer: A. Unrelieved back and flank pain.
C. Cool and pale left leg and foot.
E. Left groin egg-size hematoma.
645. Which instruction is most important for the nurse to provide a client who receives a new
plan of care to treat osteoporosis?
Answer: Remain upright after taking the medication.
646. A newly hired home health care nurse is planning the initial visit to an adult client who
has had multiple sclerosis (MS) for the past 20 years and is currently bed-bound and is lifted
by a hoist. And unlicensed caregiver provides care 8 hours/ daily, 5 days/week. During the
initial visit to this client, which intervention is most important to the nurse to implement?
Answer: Determine how the client is cared for when caregiver is not present.
647. A client with urticaria due to an environmental allergies is taking diphenhydramine...
Which complaint should the nurse identify to the client as a side effect of the OTC
medication?
A. Nausea and indigestion.
B. Hypersalivation
C. Eyelid and facial twitching
D. Increased appetite
Answer: A. Nausea and indigestion.
648. In caring for a client with a PCA infusion of morphine sulfate through the right cephalic
vein, The nurse assesses that the client in lethargic with a blood pressure of 90/60, pulse rate
of 118 beats per minute, and respiratory rate of 8 breaths per minutes. What assessment
should the nurse perform next?
A. Note the appearance and patency of the client’s peripheral IV site.
B. Palpate the volume of the client’s right radial pulse
C. Auscultate the client’s breath sounds bilaterally.
D. Observe the amount and dose of morphine in the PCA pump syringe.
Answer: D. Observe the amount and dose of morphine in the PCA pump syringe.

649. A male client is having abdominal pain after a left femoral angioplasty and stent, and is
asking for additional pain medication for right lower quadrant pain (9/10), two hours ago, he
received hydrocodone / acetaminophen 7.5/7.50 mg his vital signs are elevated from reading
of a previous hour: temperature 97.8 F, heart rate 102 beats / minute, respiration 20
breaths/minutes. His abdomen is swollen, the groin access site is tender, peripheral pulses are
present, but left is greater than right. Preoperatively, clopidrogel was prescribed for a history
of previous peripheral stents. Another nurse is holding manual pressure on the femoral
arterial access site which may be leaking into the abdomen. What data is needed to make this
report complete?
Answer: Surgeon needs to see client immediately to evaluate the situation
650. Which instruction is most important for the nurse to provide a client who is being
discharge following treatment for Guillain-Barre syndrome?
A. Avoid exposure to respiratory infections
B. Use relaxation exercises when anxious
C. Plan short, frequent rest periods
D. Continue physical therapy at home
Answer: A. Avoid exposure to respiratory infections
651. The nurse assesses a female client with obstructive sleep apnea syndrome (OSAS) who
is 5 feet tall (152 cm) and weighs 155 pounds (70 kg), the client’s 24 hour diet history
includes: no breakfast, cheeseburger and fries for lunch; lasagna, chocolate ice cream and a
cola drink for dinner, and 2 glasses of wine in the evening before going to bed for a total
caloric intake of 3500 calories. What instructions should the nurse provide? (Select all that
apply)
A. Maintain current caloric intake
B. Avoid use of alcohol as a sleep aide at bedtime
C. Reduce intake of dairy products
D. Start a weight loss program
D. Set a goal of increasing BMI (Body Mass Index)
Answer: B. Avoid use of alcohol as a sleep aide at bedtime
D. Start a weight loss program

652. A male client with impaired renal function who takes ibuprofen daily for chronic
arthritis…gastrointestinal (GI) bleeding. After administering IV fluids and a blood
transfusion, his blood pressure is 100/70, and his renal output is 20 ml / hour. Which
intervention should the nurse include in hours?
Answer: Evaluate daily serial renal laboratory studies for progressive elevations
653. The health care provider prescribes atenolol 50 mg daily for a client with angina
pectoris…to the health care provider before administering this medication?
A. Irregular pulse
B. Tachycardia
C. Chest pain
D. Urinary frequency
Answer: A. Irregular pulse
654. When obtaining a rectal temperature with an electronic thermometer, which action is
most important for the nurse to perform?
Answer: Hold the thermometer in place.
655. An adult female client is admitted to the psychiatric unit with a diagnosis of major
depressive…medication therapy, the nurse notices the client has more energy, is giving her
belongings…mood. Which intervention is best for the nurse to implement?
Answer: Ask the client if she has had any recent thoughts of harming herself
656. An adult female client with chronic kidney disease (CKD) asks the nurse if she can
continue…Medications. Which medication provides the greatest threat to this client?
Answer: Magnesium hydroxide (Maalox)
657. The nurse observes an unlicensed assistive personnel (UAP) using an alcohol-based
clean…tray to the room. The UAP rub both hands thoroughly for 2 minutes while standing at
the…should the nurse take?
Answer: Explain that the hand rub can be completed in less than 2 minutes

658. An adolescent’s mother calls the clinic because the teen is having recurrent vomiting
and…Combative in the last 2 days. The mother states that the teen takes vitamins, calcium,
mag…With aspirin. Which nursing intervention has highest priority?
Answer: Instruct the mother to take the teen to the emergency room
659. A male Korean-American client looks away when asked by the nurse to describe his
problem. What is the best initial nursing action?
A. Allow several minutes for the client to respond
B. Ask social services to find a Korean interpreter
C. Repeat the question slowly and distinctly
D. Establish direct eye contact with the client
Answer: A. Allow several minutes for the client to respond
660. An older female client tells the nurse that her muscles have gradually been getting
weak…what is the best initial response by the nurse?
Answer: Ask the client to describe the changes that have occurred
661. When organizing home visits for the day, which older client should the home health
nurse plan to visit first?
Answer: A woman who takes naproxen (Naprosyn) and reports a recent onset of dark, tarry
stools
662. A client is admitted for type 2 diabetes mellitus (DM) and chronic Kidney disease
(CKD)…which breakfast selection by the client indicates effective learning?
Answer: Oatmeal with butter, artificial sweetener, and strawberries, and 6 ounces coffee
663. A client with a postoperative wound that eviscerated yesterday has an elevated
temperature…most important for the nurse to implement?
Answer: Obtain a wound swab for culture and sensitivity
664. The nurse is reinforcing home care instructions with a client who is being discharged
following…prostate (TURP). Which intervention is most important for the nurse to include in
the client…
Answer: Report fresh blood in the urine

665. The nurse provides feeding tube instructions to the wife of a client with end stage
cancer. The client’s wife performs a return demonstration correctly, but begins crying and
tells the nurse, “I just don’t think I can do this every day.” The nurse should direct further
teaching strategies toward which learning domain?
A. Cognitive
B. Affective
C. Comprehension
D. Psychomotor
Answer: B. Affective
666. A male client with rheumatoid arthritis is schedule for a procedure in the morning.
The…unable to complete the procedure because of early morning stiffness. Which
intervention…implement?
Answer: Assign a UAP to assist the client with a warm shower early in the morning
667. The nurse is caring for a client following a myelogram. Which assessment finding
should the nurse report to the healthcare provider immediately?
Answer: Complain of headaches and stiff neck
668. A woman who takes pyridostigmine for myasthenia gravis (MG) arrives at the
emergency department complaining of extreme muscle weakness. Her adult daughter tells the
nurse that since yesterday her mother has been unable to smile, which assessment finding
warrants immediate intervention by the nurse?
A. Uncontrollable drooling
B. Inability to raise voice
C. Tingling of extremities
D. Eyelid drooling
Answer: A. Uncontrollable drooling
669. A client with multiple sclerosis (MS) is admitted to the medical unit. The client
reports…which action should the nurse implement to reduce the client’s risk for falls?
A. Schedule frequent rest periods
B. Provide assistance to bedside commode

C. Teach to patch one eye when ambulating
Answer: A. Schedule frequent rest periods
B. Provide assistance to bedside commode
C. Teach to patch one eye when ambulating
670. What is the nurse’s priority goal when providing care for a 2-year-old child experiencing
seizure…
A. Stop the seizure activity
B. Decrease the temperature
C. Manage the airway
D. Protect the body from injury
Answer: C. Manage the airway
671. A client is complaining of intermittent, left, lower abdominal pain that began two days
ago…implement the following interventions?
1. Determine when the client had last bowel movement
2. Position client supine with knees bent
3. Inspect abdominal contour
4. Auscultate all four abdominal quadrants
Answer: 1. Determine when the client had last bowel movement
2. Position client supine with knees bent
3. Inspect abdominal contour
4. Auscultate all four abdominal quadrants
672. The nurse is caring for four clients…postoperative hemoglobin of 8.7 mg/dl; client C,
newly admitted with potassium…an appendectomy who has a white blood cell count of
15,000mm3. What intervention…
A. Determine the availability of two units of packed cells in the blood bank for client B
B. Increase the oxygen flow rate to 4 liters/minute per face mask for client A
C. Remove any foods, such as banana or orange juice, for the breakfast tray for client C
D. Inform client D that surgery is likely to be delayed until the infection responds to
antibiotics
Answer: A. Determine the availability of two units of packed cells in the blood bank for
client B

673. A client with a new diagnosis of Raynaud’s disease lives alone. Which instruction should
the nurse include in the client’s discharged teaching plan?
Answer: Keep room temperature 80
674. Sublingual nitroglycerin is administered to a male client with unstable angina who
complains of crushing chest pain. Five minutes later the client becomes nauseated and his
bloods pressure drops to 60/40. Which intervention should the nurse implement?
Answer: Infuse a rapid IV normal saline bolus
675. A male client tells the nurse that he is concerned that he may have a stomach ulcer,
because he is experiencing heartburn and a dull growing pain that is relieved when he eats.
What is the best response by the nurse?
Answer: Encourage the client to obtain a complete physical exam since these symptoms are
consistent with an ulcer
676. A mother calls the nurse to report that at 0900 she administered a PO dose of digoxin to
her 4-month-old infant, but at 0920 the baby vomited the medicine, what instruction should
the nurse provide to this mother
Answer: Withhold this dose
677. When checking a third grader’s height and weight the school nurse notes that these
measurements have not changed in the last year. The child is currently taking daily vitamins,
albuterol, and methylphenidate for attention deficit hyperactivity disorder (ADHD). Which
intervention should the nurse implement?
Answer: Refer child to the family healthcare provider
678. An adolescent receives a prescription for an injection of s-matriptan succinate 4 mg
subcutaneously for a migraine headache. Using a vial labeled, 6 mg/ 0.5 ml, how many ml
should the nurse administer? (Enter the numerical value only. If rounding is required, round
to the nearest hundredth.)
Answer: 0.33 mL
Rationale:
4mg x 0.5 ml = 2/6 = 0.33 ml

679. An unlicensed assistive personnel (UAP) informs the nurse who is giving medications
that a female client is crying. The client was just informed that she has a malignant tumor.
What action should the nurse implement first?
Answer: Tell the client that the nurse will be back to talk to her after medications are given
680. The husband of an older woman, diagnosed with pernicious anemia, calls the clinic to
report that his wife still has memory loss and some confusion since she received the first dose
of nasal cyanocobalamin two days ago. He tells the nurse that he is worried that she might be
getting Alzheimer’s disease. What action should the nurse take?
Answer: Explain that memory loss and confusion are common with vitamin B12 deficiency
681. While the school nurse is teaching a group of 14-year-olds, one of the participants
remarks, “You are too young to be our teacher! You’re not much older than we are!” How
should the nurse respond?
A. “How old do you think I am?”
B. “We need to stay focused on the topic.”
C. “I think I am qualified to teach this group.”
D. “Do you think you can teach it any better?”
Answer: B. “We need to stay focused on the topic.”
682. An unconscious client is admitted to the intensive care unit and is placed on a ventilator.
The ventilator alarms continuously and the client's oxygen saturation level is 62%. What
action should the nurse take first?
Answer: Begin manual ventilation immediately.
683. After diagnosis and initial treatment of a 3 year old with Cystic fibrosis, the nurse
provides home care instructions to the mother, which statement by the child's mother
indicates that she understands home care treatment to promote pulmonary functions?
Answer: Chest physiotherapy should be performed twice a day before a meal.
684. A middle-aged woman, diagnosed with Graves’ disease, asks the nurse about this
condition. Which etiological pathology should the nurse include in the teaching plan about
hyperthyroidism? (Select all that apply.)

A. Graves’ disease, an autoimmune condition, affects thyroid stimulating hormone receptors.
B. T3 and T4 hormone levels are increased
C. Large protruding eyeballs are a sign of hyperthyroid function
D. Weight gain is a common complaint in hyperthyroidism
E. Early treatment includes levothyroxine (Synthroid).
Answer: A. Graves’ disease, an autoimmune condition, affects thyroid stimulating hormone
receptors.
B. T3 and T4 hormone levels are increased
C. Large protruding eyeballs are a sign of hyperthyroid function
685. A male client who was admitted with an acute myocardial infarction receives a cardiac
diet with sodium restriction and complains that his hamburger is flavorless. Which condiment
should the nurse offer?
Answer: Fresh horseradish
686. While completing an admission assessment for a client with unstable angina, which
closed questions should the nurse ask about the client's pain?
Answer: Does your pain occur when walking short distances?
687. A nurse who works in the nursery is attending the vaginal delivery of a term infant.
What action should the nurse complete prior to leaving the delivery room?
Answer: place the id bands on the infant and mother
688. A female client with chronic urinary retention explains double voiding technique to the
nurse by stating she voids partially, hold the remaining urine in her bladder for three minutes,
then voids again to empty her bladder fully. How should the nurse respond?
Answer: Advise the client to empty her bladder fully when she first voids
689. A client is receiving an IV solution of nitroglycerin 100mg/500ml D5W at 10 mcg/
minute. The nurse should program the infusion pump to deliver how many ml/hour? ( Enter
numeric value only)
Answer: 3 ml/hour
Rationale : 0.01 x 500 x 60 / 100 = 3

690. When assessing a multigravida the first postpartum day, the nurse finds a moderate
amount of lochia rubra, with the uterus firm, and three fingerbreadths above the umbilicus.
What action should the nurse implement first?
A. Massage the uterus to decrease atony
B. Review the hemoglobin to determine hemorrhage
C. Increase intravenous infusion
D. Check for a distended bladder
Answer: D. Check for a distended bladder
691. A-12-years old boy has a body mass index (BMI) of 28, a systolic pressure and a
glycosylated hemoglobin (HBA1C) of 7.8%. Which selection indicated that his mother
understands the management of his diet?
Answer: One whole-wheat bagel with cream cheese, two strips of bacon, six ounces of
orange juice.
Rationale:
Diet - Foods high in carbohydrates and fiber, low fat. No honey, no ham, no high sugar, no
frost food, avoid all whole wheat products.
692. Which class of drugs is the only source of a cure for septic shock?
Answer: Anti-infective
693. A 59-year-old male client comes to the clinic and reports his concern over a lump that,
“just popped up on my neck about a week ago.” In performing an examination of the lump,
the nurse palpates a large, nontender, hardened left subclavian lymph node. There is not
overlying tissue inflammation. What do these findings suggest?
A. Malignancy
B. Bacterial infection
C. Viral infection
D. Lymphangitis
Answer: A. Malignancy
694. A gravida 2 para 1, at 38-weeks gestation, scheduled for a repeat caesarean section in
one week, is brought to the labor and delivery unit complaining of contractions every 10
minutes. While assessing the client, the client’s mothers enter the labor suite and says in a

loud voice, “I’ve had 8 children and I know she’s in labor. I want her to have her caesarean
section right now!” what action should the nurse take?
A. Request the mother to leave the room
B. Tell the mother to stop speaking for the client
C. Request security to remove her from the room
D. Notify the charge nurse of the situation
Answer: A. Request the mother to leave the room
695. While caring for a toddler receiving oxygen (02) via face mask, the nurse observes that
the child’s lips and nares are dry and cracked. Which intervention should the nurse
implement?
A. Ask the mother what she usually uses on the child’s lips and nose
B. Apply a petroleum jelly (Vaseline) to the child’s nose and lips
C. Use a topical lidocaine (Zylocaine viscous) analgesic for cracked lips
D. Use a water soluble lubricant on affected oral and nasal mucosa
Answer: D. Use a water soluble lubricant on affected oral and nasal mucosa
696. The healthcare provider prescribes carboprost tromethamine (Hemabate) 250 mcg IM
for a multigravida postpartum client who is experiencing heavy, bright red vaginal bleeding.
Prior to administering this medication, which interventions should the RN implement?
A. Obtain a second IV access.
B. Decrease the room temperature.
C. Give the prescribed antiemetic.
D. Insert an indwelling catheter.
Answer: C. Give the prescribed antiemetic.
697. During the infusion of a second unit of packed red blood cells, the client’s temperature
increases from 99 to 101.6 f. which intervention should the nurse implement?
A. Stop the transfusion start a saline
B. Observe for a maculopapular rash
C. Report the fever to the blood bank
D. Give a PRN dose of acetaminophen
Answer: A. Stop the transfusion start a saline

698. An elderly female client with osteoarthritis reports increasing pain and stiffness in her
right knee and asks how to reduce these symptoms. In responding to the client, the nurse
recognizes what pathology as the cause of her symptoms?
Answer: Destruction of joint cartilage.
699. When caring for a client with traumatic brain injury (TBI) who had a craniotomy for
increased intracranial pressure (ICP), the nurse assesses the client using the Glasgow coma
scale (GCS) every two hours. For the past 8 hours the client’s GCS score has been 14. What
does this GCS finding indicate about the client?
Answer: Neurologically stable without indications of an increased ICP
700. A 46-year-old male client who had a myocardial infarction 24-hours ago comes to the
nurse’s station fully dressed and wanting to go home. He tells the nurse that he is feeling
much better at this time. Based on this behavior, which nursing problem should the nurse
formulate?
Answer: Ineffective coping related to denial
701. In assessing a client 48 hours following a fracture, the nurse observes ecchymosis at the
fracture site, and recognizes that hematoma formation at the bone fragment site has occurred.
What action should the nurse implement?
Answer: Document the extend of the bruising in the medical record
702. A client is admitted for cellulitis surrounding an insect bite on the lower, right arm and
intravenous (IV) antibiotic therapy is prescribed. Which action should the nurse implement
before performing venipuncture?
Answer: Lower the left arm below the level of the heart
703. Which assessment finding of a postmenopausal woman necessitates a referral by the
nurse to the healthcare provider for evaluation of thyroid functioning?
Answer: Cold sensitivity
704. A client with hyperthyroidism is admitted to the postoperative after subtotal
thyroidectomy. Which of the client’s serum laboratory values requires intervention by the
nurse?

Answer: Total calcium 5.0 mg/dl
705. A female client on the mental health unit frequently asks the nurse when she can be
discharged. Then, becoming more anxious, she begins to pace the hallway. What intervention
should the nurse implement first?
Answer: Explore the client’s reasons for wanting to be discharged.
706. The nurse is assessing a primigravida a 39-weeks gestation during a weekly prenatal
visit. Which finding is most important for the nurse to report to the healthcare provider?
Answer: Fetal heart rate of 200 beats/minute
707. A female client receives a prescription for alendronate sodium (Fosamax) to treat her
newly diagnose osteoporosis. What instruction should the nurse include in the client’s
teaching plan?
Answer: Take on an empty stomach with a full glass of water
708. The nurse is assessing a female client’s blood pressure because she reported feeling
dizzy. The blood pressure cuff is inflated to 140 mm hg and as soon as the cuff is deflated a
Korotkoff sound is heard. Which intervention should the nurse implement next?
Answer: Wait 1 minute and palpate the systolic pressure before auscultating again.
709. After administering a proton pump inhibitor (PPI), which action should the nurse take to
evaluate the effectiveness of the medication?
Answer: Ask the client about gastrointestinal pain
710. To reduce staff nurse role ambiguity, which strategy should the nurse-manager
implement?
Answer: Review the staff nurse job description to ensure that it is clear, accurate, and current
711. A client with pneumonia has arterial blood gases levels at: PH 7.33; PaCO 2 49 mm/hg;
HCO3 25 mEq/L; PaO2 95. What intervention should the nurse implement based on these
results?
Answer: Institute coughing and deep breathing protocols

712. The healthcare provider explains through an interpreter the risks and benefits of a
scheduled surgical procedure to a non-English speaking female client. The client gives verbal
consent and the healthcare provider leaves, instructing the nurse to witness the signature on
the consent form. The client and the interpreter then speak together in the foreign language
for an additional 2 minutes until the interpreter concludes, “She says it is OK.” What action
should the nurse take next?
Answer: Ask for a full explanation from the interpreter of the witnessed discussion
713. While assisting a male client who has muscular dystrophy (MD) to the bathroom, the
nurse observes that he is awkward and clumsy. When he expresses his frustration and
complains of hip discomfort, which intervention should the nurse implement?
Answer: Place a portable toilet next to the bed
714. A client with hyperthyroidism who has not been responsive to medications is admitted
for evaluation. What action should the nurse implement? (Click on each chart tab for
additional information. Please scroll to the bottom right corner of each tab to view all
information contained in the client’s medical record.)
Answer: Notify the healthcare provider
715. While taking vital signs, a critically ill male client grabs the nurse’s hand and ask the
nurse not to leave. What action is best for the nurse to take?
Answer: Pull up a chair and sit beside the client’s bed
716. The practical nurse (PN) is assigned to work with three registered nurses (RN) who are
caring for neurologically compromised clients. The client with which change in status is best
to assign to the PN?
Answer: Viral meningitis whose temperature changed from 101 F to 102 F.
717. An IV antibiotic is prescribed for a client with a postoperative infection. The medication
is to be administered in 4 divided doses. What schedule is best for administering this
prescription?
Answer: 1000, 1600, 2200, 0400

718. A male client notifies the nurse that he feels short of breath and has chest pressure
radiating down his left arm. A STAT 12-lead electrocardiogram (ECG) is obtained and shows
ST segment elevation in leads II, II, aVF and V4R. The nurse collects blood samples and
gives a normal saline bolus. What action is most important for the nurse to implement?
Answer: Asses for contraindications for thrombolytic therapy
719. A client with Addison’s crisis is admitted for treatment with adrenal cortical
supplementation. Based on the client’s admitting diagnosis, which findings require immediate
action by the nurse? (Select all that apply)
A. Headache and tremors
B. Postural hypotension
C. Pallor and diaphoresis
D. Irregular heart beat
Answer: A. Headache and tremors
B. Postural hypotension
C. Pallor and diaphoresis
D. Irregular heart beat
720. A client with rapid respirations and audible rhonchi is admitted to the intensive care unit
because of a pulmonary embolism (PE). Low-flow oxygen by nasal cannula and weight based
heparin protocol is initiated. Which intervention is most important for the nurse to include in
this client’s plan of care?
Answer: Evaluate daily blood clotting factors.
721. The nurse enters a client’s room to administer scheduled daily medications and observes
the client leaning forward and using pursed lip breathing. Which action is most important for
the nurse to implement first?
Answer: Evaluate the oxygen saturation
722. During a clinic visit, a client with a kidney transplant ask, “What will happen if chronic
rejection develops?” which response is best for the nurse to provide?
Answer: Dialysis would need to be resumed if chronic rejection becomes a reality

723. The nurse enters a client’s room and observe the unlicensed assistive personnel (UAP)
making an occupied bed as seen in the picture. What action should the nurse take first?
Answer: Place the side rails in an up position
724. A client is receiving continuous bladder irrigation via a triple-lumen suprapubic catheter
that was placed during prostatectomy. Which report by the unlicensed assistive personnel
(UAP) requires intervention by the nurse?
Answer: Leakage around catheter insertion site
725. A client with bleeding esophageal varices receives vasopressin (Pitressin) IV. What
should the nurse monitor for during the IV infusion of this medication?
Answer: Chest pain and dysrhythmia
726. A male client with cancer who has lost 10 pounds during the last months tells the nurse
that beef, chicken, and eggs, which used to be his favorite foods, now they taste “bitter”. He
complains that he simply has no appetite. What action should the nurse implement?
Answer: Suggest the use of alternative sources of protein such as dairy products and nuts
727. A nurse plans to call the healthcare provider to report an 0600 serum potassium level of
2 mEq/L or mmol/L (SI), but the charge nurse tells the nurse that the healthcare provider does
not like to receive early morning calls and will make rounds later in the morning. What action
should the nurse make?
Answer: Contact the healthcare provider immediately to report the laboratory value
regardless of the advice
728. Which actions should the nurse implement with auscultating anterior breath sounds?
(Place the first action on top and last action on the bottom.)
1. Place stethoscope in suprasternal area to auscultate from bronchial sounds
2. Auscultate bronchovesicular sounds from side to side of the first and second intercostal
spaces
3. Displace female breast tissue and apply stethoscope directly on chest wall to hear vesicular
sounds
4. Document normal breath sounds and location of adventitious breath sounds
Answer: 1. Place stethoscope in suprasternal area to auscultate from bronchial sounds

2. Auscultate bronchovesicular sounds from side to side of the first and second intercostal
spaces
3. Displace female breast tissue and apply stethoscope directly on chest wall to hear vesicular
sounds
4. Document normal breath sounds and location of adventitious breath sounds
729. The nurse is preparing a community education program on osteoporosis. Which
instruction is helpful in preventing bone loss and promoting bone formation?
Answer: Recommend weigh bearing physical activity

Document Details

  • Subject: Nursing
  • Exam Authority: HESI
  • Semester/Year: 2023

Related Documents

person
Charlotte Scott View profile
Close

Send listing report

highlight_off

You already reported this listing

The report is private and won't be shared with the owner

rotate_right
Close
rotate_right
Close

Send Message

image
Close

My favorites

image
Close

Application Form

image
Notifications visibility rotate_right Clear all Close close
image
image
arrow_left
arrow_right